Steven Pinker’s inflammatory proposal: universities should prioritize academics

If you haven’t yet, I urge you to read Steven Pinker’s brilliant piece in The New Republic about what’s broken with America’s “elite” colleges and how to fix it.  The piece starts out as an evisceration of an earlier New Republic article on the same subject by William Deresiewicz.  Pinker agrees with Deresiewicz that something is wrong, but finds Deresiewicz’s diagnosis of what to be lacking.  The rest of Pinker’s article sets out his own vision, which involves America’s top universities taking the radical step of focusing on academics, and returning extracurricular activities like sports to their rightful place as extras: ways for students to unwind, rather than a university’s primary reason for existing, or a central criterion for undergraduate admissions.  Most controversially, this would mean that the admissions process at US universities would become more like that in virtually every other advanced country: a relatively-straightforward matter of academic performance, rather than an exercise in peering into the applicants’ souls to find out whether they have a special je ne sais quoi, and the students (and their parents) desperately gaming the intentionally-opaque system, by paying consultants tens of thousands of dollars to develop souls for them.

(Incidentally, readers who haven’t experienced it firsthand might not be able to understand, or believe, just how strange the undergraduate admissions process in the US has become, although Pinker’s anecdotes give some idea.  I imagine anthropologists centuries from now studying American elite university admissions, and the parenting practices that have grown up around them, alongside cannibalism, kamikaze piloting, and other historical extremes of the human condition.)

Pinker points out that a way to assess students’ ability to do college coursework—much more quickly and accurately than by relying on the soul-detecting skills of admissions officers—has existed for a century.  It’s called the standardized test.  But unlike in the rest of the world (even in ultraliberal Western Europe), standardized tests are politically toxic in the US, seen as instruments of racism, classism, and oppression.  Pinker reminds us of the immense irony here: standardized tests were invented as a radical democratizing tool, as a way to give kids from poor and immigrant families the chance to attend colleges that had previously only been open to the children of the elite.  They succeeded at that goal—too well for some people’s comfort.

We now know that the Ivies’ current emphasis on sports, “character,” “well-roundedness,” and geographic diversity in undergraduate admissions was consciously designed (read that again) in the 1920s, by the presidents of Harvard, Princeton, and Yale, as a tactic to limit the enrollment of Jews.  Nowadays, of course, the Ivies’ “holistic” admissions process no longer fulfills that original purpose, in part because American Jews learned to play the “well-roundedness” game as well as anyone, shuttling their teenage kids between sports, band practice, and faux charity work, while hiring professionals to ghostwrite application essays that speak searingly from the heart.  Today, a major effect of “holistic” admissions is instead to limit the enrollment of Asian-Americans (especially recent immigrants), who tend disproportionately to have superb SAT scores, but to be deficient in life’s more meaningful dimensions, such as lacrosse, student government, and marching band.  More generally—again, pause to wallow in the irony—our “progressive” admissions process works strongly in favor of the upper-middle-class families who know how to navigate it, and against the poor and working-class families who don’t.

Defenders of the status quo have missed this reality on the ground, it seems to me, because they’re obsessed with the notion that standardized tests are “reductive”: that is, that they reduce a human being to a number.  Aren’t there geniuses who bomb standardized tests, they ask, as well as unimaginative grinds who ace them?  And if you make test scores a major factor in admissions, then won’t students and teachers train for the tests, and won’t that pervert open-ended intellectual curiosity?  The answer to both questions, I think, is clearly “yes.”  But the status-quo-defenders never seem to take the next step, of examining the alternatives to standardized testing, to see whether they’re even worse.

I’d say the truth is this: spots at the top universities are so coveted, and so much rarer than the demand, that no matter what you use as your admissions criterion, that thing will instantly get fetishized and turned into a commodity by students, parents, and companies eager to profit from their anxiety.  If it’s grades, you’ll get a grades fetish; if sports, you’ll get a sports fetish; if community involvement, you’ll get soup kitchens sprouting up for the sole purpose of giving ambitious 17-year-olds something to write about in their application essays.  If Harvard and Princeton announced that from now on, they only wanted the most laid-back, unambitious kids, the ones who spent their summers lazily skipping stones in a lake, rather than organizing their whole lives around getting in to Harvard and Princeton, tens of thousands of parents in the New York metropolitan area would immediately enroll their kids in relaxation and stone-skipping prep courses.  So, given that reality, why not at least make the fetishized criterion one that’s uniform, explicit, predictively valid, relatively hard to game, and relevant to universities’ core intellectual mission?

(Here, I’m ignoring criticisms specific to the SAT: for example, that it fails to differentiate students at the extreme right end of the bell curve, thereby forcing the top schools to use other criteria.  Even if those criticisms are true, they could easily be fixed by switching to other tests.)

I admit that my views on this matter might be colored by my strange (though as I’ve learned, not at all unique) experience, of getting rejected from almost every “top” college in the United States, and then, ten years later, getting recruited for faculty jobs by the very same institutions that had rejected me as a teenager.  Once you understand how undergraduate admissions work, the rejections were unsurprising: I was a 15-year-old with perfect SATs and a published research paper, but not only was I young and immature, with spotty grades and a weird academic trajectory, I had no sports, no music, no diverse leadership experiences.  I was a narrow, linear, A-to-B thinker who lacked depth and emotional intelligence: the exact opposite of what Harvard and Princeton were looking for in every way.  The real miracle is that despite these massive strikes against me, two schools—Cornell and Carnegie Mellon—were nice enough to give me a chance.  (I ended up going to Cornell, where I got a great education.)

Some people would say: so then what’s the big deal?  If Harvard or MIT reject some students that maybe they should have admitted, those students will simply go elsewhere, where—if they’re really that good—they’ll do every bit as well as they would’ve done at the so-called “top” schools.  But to me, that’s uncomfortably close to saying: there are millions of people who go on to succeed in life despite childhoods of neglect and poverty.  Indeed, some of those people succeed partly because of their rough childhoods, which served as the crucibles of their character and resolve.  Ergo, let’s neglect our own children, so that they too can have the privilege of learning from the school of hard knocks just like we did.  The fact that many people turn out fine despite unfairness and adversity doesn’t mean that we should inflict unfairness if we can avoid it.

Let me end with an important clarification.  Am I saying that, if I had dictatorial control over a university (ha!), I would base undergraduate admissions solely on standardized test scores?  Actually, no.  Here’s what I would do: I would admit the majority of students mostly based on test scores.  A minority, I would admit because of something special about them that wasn’t captured by test scores, whether that something was musical or artistic talent, volunteer work in Africa, a bestselling smartphone app they’d written, a childhood as an orphaned war refugee, or membership in an underrepresented minority.  Crucially, though, the special something would need to be special.  What I wouldn’t do is what’s done today: namely, to turn “specialness” and “well-roundedness” into commodities that the great mass of applicants have to manufacture before they can even be considered.

Other than that, I would barely look at high-school grades, regarding them as too variable from one school to another.  And, while conceding it might be impossible, I would try hard to keep my university in good enough financial shape that it didn’t need any legacy or development admits at all.


Update (Sep. 14): For those who feel I’m exaggerating the situation, please read the story of commenter Jon, about a homeschooled 15-year-old doing graduate-level work in math who, three years ago, was refused undergraduate admission to both Berkeley and Caltech, with the math faculty powerless to influence the admissions officers. See also my response.

179 Responses to “Steven Pinker’s inflammatory proposal: universities should prioritize academics”

  1. physpostdoc Says:

    Great piece!

  2. me Says:

    I really like the idea of basing admission on standardized test scores because that is a metric I do really well on. However I had stopped collage because the classes they require are often not things that I need. (Link to someone expressing it better) http://zackmdavis.net/blog/2012/07/trying-to-buy-a-lamp/

  3. rrtucci Says:

    I think Club Med universities with a large, carefully manicured campus are a dinosaur on an inexorable path to extinction. Your essay and Pinker’s are nice Paleontology

    I think that what will happen soon is that some university will become the McDonald’s of undergraduate education. It will have tens of thousands of franchises throughout the world. Most of the coursework will be done online, through carefully optimized MOOCs. Students will meet maybe once a week in a small facility (maybe their existing high school or vocational school) to take tests in the presence of a proctor and to take the occasional lab course or manual training.

    MacUni will give as good an education as MIT at 1/20 th the price. Soon MacUni will start attracting people as smart and gifted as an MIT, except in much larger numbers due to its having 1,000 times as many students

    I think that this is inevitable because half the population of world earns less than 3 dollars a day but they deserve an MIT education (if they want it and can hack it) as much as you or me.

  4. Ewout ter Haar Says:

    I understand the frustration with the current system, but it seems to me that the “only standardized tests” admission system would be even easier to game by wealthy parents, leading to “iron law of meritocracy” dynamics. You can see this in action in countries like India or Brazil. Attempts, however ineffective, to remedy this injustice must be made, right?

    However, the larger point is that both you and Pinker assume the existence of gatekeepers to higher education. Wouldn’t it be a worthwhile exercise to imagine a system where there is less need to restrict access to higher education and where universities can’t derive part of their value because of their country-club character?

  5. D Says:

    There’s something that Pinker mentioned in his article that you left out here: Harvard doesn’t want people like you as undergrads! They’re explicitly not looking for future professors; they want future rich/powerful people. Their current admissions procedure probably does do a better job of finding those people than relying on standardized tests would. For places like MIT, Caltech, UChicago, etc, focusing on tests makes more sense, and I think they already use them more heavily than Harvard does. But I guess not enough, if they all rejected you.

  6. Scott Says:

    rrtucci #3:

      I think Club Med universities with a large, carefully manicured campus are a dinosaur on an inexorable path to extinction.

    It’s funny: pundits have been saying that for as long as I can remember. Yet somehow, the high-school students on the ground seem not to have gotten the memo about the inevitability of this historical change. On the contrary, more and more of them apply to elite universities, and they jump through more and more hoops to try to get in, forcing the universities to find more and more grounds for rejecting them. When do you foresee your MacUni starting, how will it draw applicants away from the Ivies, and why hasn’t it happened already?

  7. Scott Says:

    Ewout #4:

      However, the larger point is that both you and Pinker assume the existence of gatekeepers to higher education. Wouldn’t it be a worthwhile exercise to imagine a system where there is less need to restrict access to higher education and where universities can’t derive part of their value because of their country-club character?

    Yes, let’s imagine! I’m a huge fan of democratizing access to higher education—and I’d say that the web, Wikipedia, the arXiv, online course notes, MOOCs, Khan Academy, and so forth have already done that to an extent that would’ve been hard to imagine 25 years ago.

    At the same time, I’m wondering about the brass tacks of your proposal: should elite residential universities like Harvard and Princeton cease to exist? If not, then it seems to me that well into the foreseeable future, they will get way more applicants than they can physically accommodate, so they will need to reject most of them, and the only question will be on what grounds.

  8. not lobos motl Says:

    I’d say you’re underestimating the extent to which admissions committees already rely on test scores. I’m sorry 15 year old Scott didn’t get into Harvard, but Harvard gets lots of applicants with perfect SAT scores who don’t have “spotty grades.” If you really want to have a go at college admissions committees, why not criticize the preference for “legacy applicants” or the flagrant bribery involved in admitting kids whose parents make large donations to the university? These practices seem far more pernicious than awarding a few points for an interesting, thoughtful application essay.

    Also, my view of testing might be more favorable if universities didnt outsource testing to a private monopoly. This is really the problem with standardized testing, generally.

  9. Elizabeth C Says:

    The same textbooks are available in every university library, so the primary reason for undergraduates to attend an elite university is to network with their peers. A student who is self-motivated to master the material may in fact be more likely to thrive at a “lesser” school, where they can receive more personal attention from professors and are less likely to have “school interfere with their education.”

    The academic value of being at an elite institution goes way up when the student transitions from textbook learning to doing research, which ideally coincides with transitioning to grad school and the corresponding chance to be reassessed. Overall I think the system works pretty well, but maybe we need to create better opportunities for early bloomers who are eager to specialize in research.

  10. Noah Says:

    I don’t really understand why none of the elite colleges took the opportunity to grow as the number of applicants grew enormously. Why didn’t Harvard open a California campus and a Texas campus? Why did every last one of them decide to become way more selective over the past 40 years instead of staying equally selective and growing?

  11. pierre menard Says:

    Possibly relevant to your argument: concerns about SAT scores being a proxy for income through the use of test preparation services are largely without foundation. See a summary of the empirical literature at

    http://marginalrevolution.com/marginalrevolution/2014/03/the-sat-test-prep-income-and-race.html

  12. Scott Says:

    Noah #10: There are plenty of state schools that have opened branch campuses, but what usually happens is that the branches get considered “less prestigious” than the main campus, rather than inheriting the latter’s reputation. I’m guessing the same would happen if Harvard opened a branch campus. Still, it’s a very interesting proposal—and I’m happy to say that my own alma mater has taken it up, opening Cornell Tech in NYC (jointly with the Technion).

  13. Douglas Knight Says:

    1. Yes, standardized tests were intended to democratize, but not for the purpose of recruiting immigrants, but for the purpose of recruiting rural youths.

    2. Putting MIT and Harvard in the same breath is ridiculous. Consider this graph (this source).

    3. Which is not to say that you should not scold MIT just because it is ahead of the pack; in fact, it is probably easier to reform MIT because it is clearer what it is doing and what it is trying to do. Your view of Harvard admissions is a common one, but I am not convinced it is true. Harvard claims to admit “special” applicants, not well-rounded ones. It is a lot more plausible about Harvard than about Yale.

  14. Scott Says:

    not lobos motl (but lubos motl?) #8:

      If you really want to have a go at college admissions committees, why not criticize the preference for “legacy applicants” or the flagrant bribery involved in admitting kids whose parents make large donations to the university?

    Err, I did criticize it (see the last sentence of the OP). In my view, such bribery can only be justified if the financial benefit to the other students exceeds the moral cost of the university’s prostituting itself in that way. And if, as with some elite universities, the alumni donations just get squirreled away in an already-massive endowment, or spent on superfluous building projects, rather than going toward financial aid for the poorer students, then it has no moral justification at all.

      These practices seem far more pernicious than awarding a few points for an interesting, thoughtful application essay.

    I agree, but I think that comment is disengaged from the reality on the ground. Personally, I’d love nothing more than to judge applicants by the quality of their writing. The trouble is that what’s being judged is less the writing itself, than whether or not that writing documents a high-school career spent in the appropriate sort of political campaign to get in to an elite college. In any case, application essays are extremely likely to have been ghostwritten, or at least extensively edited by parents and coaches. For that reason alone, a writing test (whatever its problems) seems preferable.

  15. gasarch Says:

    Some random points
    1) High Schools should also de-emphasize sports. Many High Schools have a trophy case displaying their sports victories but nothing about their academics. This might baffle other countries. It baffles me!

    2) Teaching-to-the-test is terrible for the SATs but great for Math competitions (you learn some real math). I think that having a test that is hard enough so that teaching-to-the-test is still teaching you something, but not quite so hard as a math competition works.

    3) I would take HS grades, or at least HS courses into account (e.g., if they took lots of AP courses, that says something)

    4) Harvard might still get 20,000 student with perfect scores on standardized tests. How to break ties and you’re back to ways to game the system.

    5) Some HS students do X to get into college, and others do X because they really care (X could be, say, helping the homeless). Very hard for admissions committees to tell.

    6) Not sure what Scott’s `special qualities for a minority of students’ is and how it differs from the extracurccilars people are expected to do now.

  16. Scott Says:

    gasarch #15:

    I’m in strong agreement with 1) and 2) (though I’m skeptical that if someone improves their SAT score by a few hundred points, they didn’t have to learn anything good in order to do so).

      Harvard might still get 20,000 student with perfect scores on standardized tests.

    If so, then the obvious solution is to make the test harder! (E.g., I haven’t heard of a problem with too many students getting perfect scores on the Putnam. 🙂 )

      Not sure what Scott’s `special qualities for a minority of students’ is and how it differs from the extracurccilars people are expected to do now.

    You supplied the answer to your own question! The special qualities are for a special minority of students.

  17. Shmi Nux Says:

    Many places in Asia and Eastern Europe use entrance exams, with the more “elite” universities designing harder ones. Is there any evidence that their alumni are better than those from the universities using a US-style admission process? Obviously there are plenty of confounders, but I wonder if they can be controlled for, at least in part.

  18. a random thought Says:

    It appears that whenever college admissions is discussed, especially into “elite” institutions, accepting children of those who write fat checks is treated as somewhere between a mortal sin and a necessary evil. I feel it stems from a middle-class-centric perspective of treating colleges as vehicles of attaining (and retaining) socio-economic success.

    Note however that, unless a 99% estate tax is instituted, *children of billionaires will be billionaires* whether they go to Harvard or not. Wouldn’t it be better to expose them for four years to what real science and intellectual pursuits are about? Even if they spend most of their time partying, something is bound to rub off.

    It this translates into the next generation of billionaires funding the next Perimeter Institute instead of buying a Football team, or if they are “just” a little more enlightened in running their inherited empires, wouldn’t it have been worth the admissions spot?

  19. rrtucci Says:

    “how will it draw applicants away from the Ivies,”

    The Ivies will join the MOOC oil rush too, because greed is a very powerful human motivator. Remember Titusville, ye Pennsylvanian Aaronson 🙂

    What makes more money, a French restaurant that caters to a small, local, affluent clientele, or a world wide chain of MacDonalds (or Starbucks) sucking in people from the whole economic spectrum?

  20. Shub Says:

    I went through school, college and beyond only based on grades. The is no way I could have done it otherwise.

    That’s bad preparation too – as the outside world doesn’t run that way.

  21. Matt2977 Says:

    Scott,

    Some questions:

    Is there a standardized test for philosophical ability?

    For intellectual and ethical depth?

    For creativity?

    For the ability to see through calcified ways of thinking about the world?

    For curiosity?

    Are artistic and musical abilities not a form of brilliance?

    I was thrown repeatedly by Pinker’s essay. In several places, he criticizes people for employing anecdotes to make their cases. And then he pulls out his anecdote about the “skilled professional” friend of his, and her son’s “community service” project.

    He engages in false-choice fallacies, too. Apparently students can either “seek affluence, success, and prestigious careers” or else “smoke weed and play video games on their parents’ couches.” What about meaningful careers that quietly improve the lives around us, make important but incremental steps toward solving difficult societal problems, cause us to reflect about the meaning of our existence, break down conventional assumptions about the world, generate important new ideas or ways of seeing the world, or create moving art and music?

    Apparently there are just two kinds of students: those who “try to get an A in every class,” and those who “turn in shoddy work.” What about students who work hard, do a careful and thoughtful job, but aren’t grade-grubbers when they don’t get their way?

    And, please: “bohemian authenticity” versus “worldly success and analytical brainpower”? That’s it? Just put us all on Adderall why don’t you.

    Actually, I wouldn’t be surprised if Pinker regarded that as a fine idea. After all, if our top value is just worldly success and analytical brainpower in the raw, why not pump us full of any drug that increases those metrics?

    There’s plenty of “dogmatic assertion” in Pinker’s piece, and not as much objective analysis as one might have supposed. Sure, there’s the familiar fetishization of data points that looks like objective analysis, and a breathtakingly naive faith in the rigor of sociological studies and surveys. (Just like a psychologist, amirite?)

    This isn’t math or physics. And yet, just think about how mind-crushingly hard it is to extract the Higgs from particle physics data. And that’s orders of magnitude simpler than the human/societal systems in question here!

    Pinker, of all people, should recognize just how staggeringly difficult it is to extract causation from correlational sociological data! Not to mention how easy it is for someone with a strong bias (Pinker has long been biased toward certain ways of thinking about innate human ability) to see what they want to see when looking at statistics.

    And not everything is reducible to data, either. Elections, sports statistics, etc. — sure. But subjective judgment is an unavoidable part of how we evaluate the most important things in our lives, from our morals to our values to how we decide what success even means in the first place. (Is there a place in Pinker’s cosmology for learning about those things in college?)

    Pinker does something else that drive me crazy: He declares that because he doesn’t understand what it really means to think, then no such meaning exists. But come on — maybe a school of engineers is some person’s idea of paradise, but to a lot of intelligent people, there’s just something missing in an environment like that. There’s something off about Caltech, and even MIT, that drives away a lot of very intelligent prospective undergrads.

    It’s a definite lack of humanity that one sometimes experiences in places like those.

    What is humanity, you might ask? It’s not very easy to define — but, then again, neither is consciousness, or pornography. And yet, to paraphrase Potter Stewart, we know it when we see it.

    The truth is that there are human qualities that are valuable and worthy of cultivation, and that are not so easily testable. Humility is one of them.

    I was fascinated to read through Pinker’s list of what a university should be teaching young people. There’s plenty of science and math, descriptive anthropology, political science, and history. My favorite is that “They should be acutely aware of human fallibility, most notably their own.” (Doctor — heal thyself!)

    That he would actually write of musicians and artists at top schools that “it’s not clear why they could not have had the same experiences at Tailgate State, or, for that matter, the local YMCA” speaks to a startling lack of both awareness, exposure, and thoughtfulness on Pinker’s part. Don’t you think there’s some rational reason why the world’s greatest young musicians and artists might want to be in the same place?

    And finally, as to testing, I want to get us clear on this right now. There is a well-known discrepancy between the performance of certain racial or ethnic groups (or women vs. men) on standardized tests. Do you believe these are “real” effects — i.e., genetic, like Pinker keeps saying with repetitive explicitness — or not? Are you in the Charles Murray/Larry Summers camp? What do you think would happen to the ethnic, racial, and gender diversity of campuses if we relied solely on standardized testing?

    There’s a certain lack of thoughtfulness running all the way through Pinker’s article. He declares that Deresiewicz is wrong to say that the SAT “measures” affluence, because it merely “correlates” with parental income. But that’s a measurement, for heaven’s sake! All measurements consist of creating a correlation between things.

    Apparently “Matt McGue has shown, moreover, that adolescents’ test scores track the SES only of their biological parents, not (for adopted kids) of their adoptive parents, suggesting that the tracking reflects shared genes, not economic privilege.” So that’s it! the 50-50 mix of nature vs. nurture is out the window because of one guy named Matt McGue! Just brilliant!

    Pinker asserts that universities would be crazy to look for graduate students the way they look for undergraduates. Perhaps he’s right when it comes to dance and rowing, although I’d argue that art and music PhD programs are pretty darn keen on getting students who are great at art and music.

    But could you imagine selecting PhD students just based on standardized tests? If you ask people who run admissions for science/math PhD programs, do you think they’d tell you that a standardized test (let alone the subject GREs!) could ever be more than a tiny part of the application? No. They’ll tell you that they want to see grades, research experiences, recommendation letters, and numerous other “soft” qualities that provide a window into the student’s depth, creativity, enthusiasm, knowledge, competence in the lab, and work ethic.

    As one of the deepest human beings I know of, I’m actually a little surprised at just how much you seem to like Pinker’s essay!

  22. Yinon Raviv Says:

    I think we can all agree that the college application process needs some serious re-thinking. It needs innovation, which means it needs some risk taking. And while you make the point that standardized tests democratize the process and reemphasize academics, I still hold the belief that as is, standardized tests hurt more than they help. Give me a good standardized test that test critical thinking, logical reasoning, and creativity effectively and I’ll reconsider.

    My crackpot ideas are the following:

    – Beef up the interview process. This would take an insane amount of money, infrastructure, and time, but guess what? Face to face interactions are as comprehensive as they can get. When colleges say they’re looking for “intellectually curious” or “cause-driven” or “leaders”, they can add real meaning to it. Yes, it’s just like a job interview. Colleges preparing kids for the real world? Oh no….
    – Have a process where the student submits some of his best work in the classroom, signed off by a teacher. If it’s an English/Humanities major, have him/her send their best essay they’ve written, and make them explain why it’s so special to them. Physics/engineering, make them send a project. Having the teacher sign off on these submissions would represent a MUCH better reflection on how ready the student is for college, or at the very least, how academically strong the student is.
    – My wildest idea: put 30-50 applicants in a room with either live proctors or cameras. Present challenges to the room and observe the applicants working with their peers. The challenges need to be comprehensive, intentional/specific, and I’d say collaborative. Comprehensive so you can see students display an array of skills and qualities, be it taking on the role of a specializer or a presenter or a group manager or a support role, whatever it may be, don’t make it simple to emphasize one kind of person. At the same time, the college needs to realize what they’re looking for… some colleges love the front of the room, rahrah leaders with great interpersonal skills, and others highly value the inquisitive and curious. Finally, unless the college specifically wants the most competitive and cutthroat sharks in the country, make the tasks/challenges collaborative (and not just a bullshit “group project”). Again, I’m just kicking ideas around. If I had my own university on my own massive endowment, I’d invest my hypothetical riches in it.

    That was a lot to read, and it’s just me floating random crazy ideas. Felt good.

  23. Michael Dixon Says:

    I feel obligated to leave this here:
    http://www.youtube.com/watch?v=Y-OLlJUXwKU

    It’s a short segment by Bloomberg that gives you a glimpse as to how Amherst College make their admission decisions. Warning: You will (should) be appalled.

    — — —

    “I was a 15-year-old with perfect SATs and a published research paper, but not only was I young and immature, with spotty grades and a weird academic trajectory, I had no sports, no music, no diverse leadership experiences. I was a narrow, linear, A-to-B thinker who lacked depth and emotional intelligence: the exact opposite of what Harvard and Princeton were looking for in every way.”

    @Scott:
    Sure, you might not have been a complete package from that perspective, but these are just qualifications of an ideal and “deserving” student (their perspective). I guess they tend to call this a “fit”, though I see it as a way to play it safe. What these institutions should have considered was your potential and whether they are suited to help you achieve/surpass it. This includes opportunities to develop what you might have comparatively lacked. Perhaps that’s what Cornell and Carnegie Mellon were able to accomplish, whereas the others did/could not?

    I could jump in on the disccusion and highlight the ways I think the system is problematic, but I won’t here. I’m be more interested to hear the rest of your story. What happened after getting rejected from all those universities and going to Cornell? What special things there allowed you to overcome the problems (+barriers) in the past and thrive? What do you imagine would be different if you had been accepted into those other institutions?

  24. Evan Says:

    The way you make the ‘special requirements’ special is by putting a quota on the number of people to which they apply. So if you are in the top 100 “most interesting” people that apply to Harvard, you get in. Otherwise, that information gets thrown out and you are judged on your measurable qualities — you don’t get bonus points or win tie breakers for having extra-curricular activities. Once high scores are the “easy” way to get in, people won’t be competing to have special life experiences just to get into college, and hopefully your will be able to fill those slots with people who are actually interesting.

    Not sure I agree with the grades thing. It is appealing to me because I had great test scores and spotty grades as well, but I also recognize that “test taking” is a skill, and one at which I am extremely good. Grades tell you something different than test scores, and it is a quantitative measurement that should be predictive of success in college. Ignoring them seems like throwing away real information. If grades really are too variable and meaningless to trust, that is just another reason we should be fixing our public schools. Honestly though, I can’t believe that in a world where google seems to know what I am about to search for before I start typing, we can’t find a way to standardize grades into a meaningful number that correlates with results. It won’t be perfect, but admissions don’t need to be perfect.

  25. GradingIsGreat Says:

    Intersting thing on high school grades:

    Even IF you assume the following premises (which are obviously false)

    1. High school grades are unbiased
    2. Grades dont vary from school to school
    3. Grades are based on actual criteria

    EVEN THEN, you get a SINGLE DIGIT of grade. That is, you can distinguish between a 2 GPA and 3 GPA, but not between a 3 and 3.5. Its like trying to measure the height of someone with a yardstick that has no markers.

    The reason for this is that grades are broken into 10 bins, and coarse grained heavily. You lose a huge amount of information when you do that, and the number of digits you get to use is basically the square root of the number of bins.

  26. Darrell Burgan Says:

    The unfortunate part of standardized tests is that the entire American school system simply becomes a machine for teaching kids how to best those tests, rather than focusing on genuine education. The problem is that the standardized tests cannot test on all facets of human knowledge. Even within the subject areas (math, for example) they focus on a significant subset of the possible things students could be learning. The inevitable result is a school system that teaches students only what is on the tests, and eschews any other learning that is arguably of equal value.

    I agree that standardized tests may be as good as it gets with current educational science. But the state of the art can, and must, improve.

  27. Petter S Says:

    All Swedish universities admit the majority of their students based on high-school grades.

    A couple of years ago, the school system was deregulated and private schools were allowed and they were allowed to make a profit.

    What do you think happened? You don’t have to be an economist to guess “inflation.”

    I wish universities here would admit more students based on the standardized test.

  28. Sam Tobin-Hochstadt Says:

    Pinker’s essay was good for the first two-thirds, but then it veers into genetic determinism and blithe ignorance about some of the consequences. Sadly, you end up endorsing the latter. Hopefully you don’t actually think that eliminating black, Latino, and working-class representation at Harvard and MIT would be a good thing.

  29. francis Says:

    I am not very familiar with the educational system in the US but is it really the case that the elite universities give you a better education than say universities like Cornell? And if yes, by what measure?

    Maybe being around lots of shallow people who optimize their CVs to ridiculous criteria is worse than being around people with real passions however nerdy and specialized they may be?

  30. domotorp Says:

    I agree with most of what you say but let me add that the situation at other parts of the world could be also improved. In many countries that inherited the British education system ranks (e.g., against classmates) matter more than scores what can create a very unfriendly competition in class. In some other countries (like in Hungary the high schools) the exam is so standardized that classes specializing in a subject have limited freedom to admit students who excel at their field, like math or arts. But probably you are right that US is one of the worsts.

  31. Peter Says:

    I would do a retrospective study of those graduates doing well now to find out what got them in, say, 20-30 years ago.

  32. Haelfix Says:

    Similar trajectory to Scott here. Nearly perfect SAT scores (missed 1 question somewhere), 15 when I graduated, nearly perfect grades at an elite highschool, strong olympiad showings, science fair showings, really good recommendations etc. I was even semi ok at sports.

    Yet I was still rejected at nearly every Ivy league and ended up going to a perceived lesser school, only to be pursued by all the big Ivies as a grad student and later in academic life.

    I remember tutoring 18 year olds in AP tests and SATs who scored far below me who ended up going to Harvard/Princeton/Yale as undergrads etc.

    It was instantly clear the system was rigged away from education. I wasn’t a woman/minority or a legacy, I had no sports credentials and my half international (European) heritage was seen as a negative.

    A strange world indeed.

    I agree with Scotts premise and am in favor of test scores/meritocracy ultimately, although I would like to see some sort of extra tests to distinguish the right end of the spectrum. I always felt like the Baccalaureate and Olympiads were much better tests than the ridiculous SAT (why not simply cull based on IQ). Especially the latter where the correlation between being a winner and success in the hard sciences is really strong.

  33. Scott Says:

    Shmi Nux #17:

      Many places in Asia and Eastern Europe use entrance exams, with the more “elite” universities designing harder ones. Is there any evidence that their alumni are better than those from the universities using a US-style admission process? Obviously there are plenty of confounders, but I wonder if they can be controlled for, at least in part.

    Yeah, the confounders are what I worry about. It could be—and I’d guess, probably is—that alumni of American elite universities do extremely well despite what Pinker calls the “eye-of-newt-wing-of-bat” admissions process.

    However, one data point might come from PhD programs in the hard sciences, where American universities do indeed now take the majority of their students from abroad—with heavy representation from countries like India, which admit to the IITs based on test scores. Though once again, there are many possible confounders here, including the huge population size of India (and more generally, of the complement of the US), and the fact that lots of top US undergrads prefer lucrative Wall Street or Silicon Valley careers to going to graduate school. The perils of social science research.

  34. Scott Says:

    Matt 2977 #21:

      Some questions:

      Is there a standardized test for philosophical ability?

      For intellectual and ethical depth?

      For creativity?

      For the ability to see through calcified ways of thinking about the world?

      For curiosity?

      Are artistic and musical abilities not a form of brilliance?

    No, I don’t think there are good standardized tests for any of those things, and I fully agree with you about their importance. But once again, you fail even to ask the key followup question:

    So how do you propose to measure those qualities in 17-year-olds? What’s your alternative evaluation method? Should admissions officers be trying to peer into the applicants’ souls, to look for ethical depth? As I said, that’s exactly what they try to do today, and we now know that it produces a tragicomic result, with a giant industry devoted to manufacturing the semblances of creativity, intellectual and ethical depth, etc. for the purpose of getting in to Ivy League universities. For me, any response that fails to grapple with that empirical fact, might as well be discussing the admissions policies on Planet Zorg.

    As I said, my solution is: allow a minority of students in for genuinely special reasons (i.e., they achieved something amazing that no test could capture), but never let “specialness” become the main path to acceptance—because as soon as you do, specialness will simply become another gamable commodity, and you’ll be worse off than if you’d simply relied on tests.

  35. jk Says:

    “RETURNING extracurricular activities like sports to their rightful place as extras”

    when exactly was focusing on academics [for undergraduates] the central feature of admissions?

    iirc, these institutions were originally for the training of clergy, but at over the course of the 18th century transitioned to the perpetuation of elites. “building character” and “liberal education” were fine goals, since students were already members of the power elite, whose futures were guaranteed not by their academic achievements or by learning any particular skills, but by their families’ place in society.

    the introduction of ANY academic criteria was a step towards making academics a focus of these institutions. but to talk of “RETURNING” to a focus on academic achievement for undergraduate admissions implies that such a state of purity existed at some time in the past. when was that, exactly?

  36. eitan bachmat Says:

    Here are a few comments from the point of view of an outsider. These comments do not lead to a grand unified theory of college admissions, just some sporadic and not necessarily correct observations.

    One great difference between the American college admissions system and that of say, the Israeli (German) or French system is
    that in these other systems you are not accepted to a university, instead you register to a certain major (study program) and are accepted to that study program which is provided by one of the university departments using some weighted criterion involving standartized tests and some HS scores (the French system is most extreme in this sense, relying on a national competition). Consequently, we dont really have prestigious universities, but rather some (very few) departments which are hard to get into, as a result of high demand and few spots. For example, you could have been a complete imbacile, with no talent at all (like me) and study math (or math/physics) at HUJI. When I was a student, you needed a D average in HS to get in, since there was little demand (this hasnt changed much). Here are some of the teachers that potentially would have taught you, Michael Rabin, Nati Linial, Alex Lubotzki, Hillel Furstenberg, Robert Aumann, Joram Lindenstrauss, Shimshon Amitsur, Aryeh Dvoretzky, Saharon Shelah, Gil Kalai and Peter Sarnak (as a guest lecturer) to name a few in no particular order (Avi Wigderson joined a bit later)…, some of your potential schoolmates could be, Oded Schramm, Itai Benjamini, Noam Nissan, Muli Safra, Ran Raz, Yossi Azar and many others, while the graduate students would have included Noga Alon. Not such a shabby list in my opinion.
    I wanted to start studying part time for credit while I was still in high schhol. Since I didnt have a high school diploma or SAT scores, I was called to an interview with the department head, the late Alex Zabrodsky (an algebraic topologist who was once a member of Israel’s most selective entertainment troup, Lehakat Hanahal, thats what I call multi-talented). Prof. Zabrodsky asked me why I wanted to study. I said I liked math. He gave me two books to read, a calculus book and a linear algebra book and said that if I can deal with these, I will be OK. That was the acceptance process. In many ways it still is for many Israeli teenagers who start college before finishing high school, including many of the Israelis you knowfrom your favorite CS topic.
    Other Israelis will start school after the army, but again, it is not particularly difficult to get accepted.

    This meant that I could spend my childhood playing basketball and soccer (for fun, not to make college acceptance easier), watch a lot of junk TV and artsy movies, read about many things, listen to a lot of rock music, in short,
    become well rounded before college and have a normal and happy childhood without any participation in official programs of any sort. Also tuition was low (its now 3000$) and since I lived in Jerusalem, I didnt have to go somewhere else for college (Israel is a tiny country, and you can almost always find a terrific university nearby). My parents were never involved in the process in any way, shape or form, they just came to graduation (see Pinker’s article for context).

    My (non) experience with the American college system
    comes from a sabbatical that we spent in the Boston area.
    My elder son was in 9th grade. He actually enjoys competitive sports and joined a local swimming team. Many of the kids on the team were pretty good at swimming but didnt like it at all. They swam since it gave them a competitive edge (lower SAT scores for addmision) and a letter of recommendation. Many did get into very good schools and immediately following admission stopped swimming till college.
    My son had tremendous difficulties with written English. He is atrue intellectual that just likes learning so he was very good in history tests. He has also seen many of the places that were discussed, but he got a D because he couldnt submit written summaries of the various chapters. The teachers explained to us that while 9th grade scores would not enter school records for college (10th grade would), this deficiency will still blemish his record and that we should (with school knowledge) do his HW for him. Being Israeli we obviously told them to %$@# off, but in general dealing with school involved a lot of anxiety, my son also was given an IQ test and scored a 90 (well below average), explanations that he couldnt read English didnt help.
    I cant imagine the stress of 3 more years like that, in the hope of getting into a reasonable (not even prestigious) college. Luckily, we went back to Israel.
    I am happy to report that my (officially IQ deficient) son graduated cum laude in math, with a research paper level senior thesis (proving Hilbert’s irreducibility theorem using Weyl’s polynomial equidistribution theorem) at the age of 19 in a non prestigious Israeli university, while spending most of his time on true extracurricular (non credit) activities that made him very well rounded. I expect him to do very well in his professional life whatever it may be.

    Following these experiences, what I really dont understand is why Scott, as a 15 year old, you even bothered with the American system, you should have come to Israel for a good, inexpensive and fun college experience, with essentially automatic acceptance.

  37. Scott Says:

    Matt #21:

      But come on — maybe a school of engineers is some person’s idea of paradise, but to a lot of intelligent people, there’s just something missing in an environment like that. There’s something off about Caltech, and even MIT, that drives away a lot of very intelligent prospective undergrads.

      It’s a definite lack of humanity that one sometimes experiences in places like those.

    I’m reminded of what Richard Feynman, one of history’s most famous Caltech profs, replied when someone told him that he (Feynman) proved that “physicists can also be human”: “I am human enough to tell you to go fuck yourself.” 😉

    In 7 years teaching at MIT, I have yet to notice any lack of humanity among the students here, let alone a “definite” lack.

    Sure, tech schools like Caltech, MIT, CMU, and Harvey Mudd do have a “vibe” that drives certain students away, but so do “preppy” schools like Princeton and Yale. To each her own.

    And social effects can easily run the other way: living at Telluride House at Cornell, I was surrounded by literary theorists, people liable to use words like “scientism,” “simplistic,” or “hegemonic” in a sentence. The result? I felt “something missing in an environment like that,” and one of the only other math/science people in the house became my lifelong friend.

  38. ks Says:

    I think one way to make the tests like SAT to be game-proof is not to reveal the structure of the test. So the test format, and even the test time shall differ each year. So no one will know what the test will look like in any given year, except that it will have reasonable questions from the specific subject. And the questions shall be of type that tests the real understanding of the concepts, mostly of descriptive types probably. This will likely favor the students who learn for the sake of learning, and eventually that might become the norm. While it might require more effort to prepare tests like this, and grading the test papers, it is not impossible.

  39. The_Duck Says:

    Crucially, though, the special something would need to be special. What I wouldn’t do is what’s done today: namely, to turn “specialness” and “well-roundedness” into commodities that the great mass of applicants have to manufacture before they can even be considered.

    Oh, come on. This is what admissions committees already try to do. Why do you think you could do it better? As you point out earlier in your post, *anything* you choose as an admissions criterion will instantly become a “commodity.”

  40. Darrell Burgan Says:

    Scott #34: you said:

    So how do you propose to measure those qualities in 17-year-olds? What’s your alternative evaluation method? Should admissions officers be trying to peer into the applicants’ souls, to look for ethical depth?

    I propose that this an unsolved problem in education science, and education scientists should urgently pursue it as an important line of study. I see no reason that an impartial assessment of all of those qualities cannot occur, whatsoever, given a significant enough sample size and a reasonable set of error bars.

    I work with a data science group that has developed a 30+ dimensional model that can predict how closely incoming employees align to the culture of the organizations that they are joining, and we have empirical facts that show using such a model cuts turnover by up to half and improves performance by by up to a quarter, depending on the organization. It’s not perfect but it is the state of the art in HR science. Why cannot education scientists accomplish the same?

    My suspicion is a lack of emphasis on the subject. I am fully in favor of admissions being merit-based. I am highly suspicious of any statistical merit models that have small numbers of dimensions and variables.

  41. fred Says:

    Spent 5 years in engineering polytechnic school in Europe (now officially the US equivalent of a combined BS+MS).
    Actually that was the only school/college/university in the country to have an entrance exam (multiple sessions on geometry, algebra, calculus).
    Even with that, a lot of the students would fail in the first year or second year – the program was very intensive, we’d be in school from 8am till 8pm, toiling like maniacs (theory in the morning and practical labs in the afternoon). There was very little room for creativity in the first few years.

    I then moved to the US and completed an extra MS in engineering at an Ivy school (had to take the GRE, etc). The big surprise was that most of us were foreigners, almost no Americans.

    My only experience with BS in the US is watching my nieces and nephews. I’m always quite shocked at how “aimless” and “relaxed” those programs seem to be in comparison to what I went through at their age back in Europe.

  42. Scott Says:

    The_Duck #39: As I said before, the difference is that I would strictly cap the number of applicants admitted for “special” reasons, to prevent “specialness” from becoming the mass-produced commodity that it is today.

  43. Vadim Says:

    Darrell,

    One issue I see is that since so few companies currently use your model, job applicants aren’t trying to game the system. If your model was widespread enough for job applicants to take it into account, I question how effective it would be.

  44. lewikee Says:

    Just wanted to highlight the importance of the distinction between public and private institutions:

    I don’t see why private universities “should” change anything. They can make it their policy to only admit legacies that work full-time in soup kitchens for all I care. They answer only to themselves. However, they will have to deal with the resulting reputation. But for public institutions receiving public money with the public understanding that academics is to be made a priority, then I agree that things need to change; since it is then an issue of money taken for work not done.

  45. Observer Says:

    See Pinker rebutted here: “More than 80% of admissions at Harvard (and other elite schools like Williams) is determined by academic merit, measured by past success in high school (high grades in the most rigorous classes with the best teacher recommendations and top standardized test scores), all of which best predicts academic success in college.” What Pinker Gets Wrong About Harvard (and Williams) Admissions

  46. Rand Says:

    Some people would say: so then what’s the big deal? If Harvard or MIT reject some students that maybe they should have admitted, those students will simply go elsewhere, where—if they’re really that good—they’ll do every bit as well as they would’ve done at the so-called “top” schools. But to me, that’s uncomfortably close to saying: there are millions of people who go on to succeed in life despite childhoods of neglect and poverty. Indeed, some of those people succeed partly because of their rough childhoods, which served as the crucibles of their character and resolve. Ergo, let’s neglect our own children, so that they too can have the privilege of learning from the school of hard knocks just like we did. The fact that many people turn out fine despite unfairness and adversity doesn’t mean that we should inflict unfairness if we can avoid it.

    This is weak. “Justice” does not demand that top universities admit students according the criteria preferred by the top mathematicians. These are not disadvantaged individuals, these are individuals with incredible advantages. To look even at my own situation, that I was able to quit a job in tech to pursue a PhD at a top university and get paid for studying there, is nothing short of absurd, in a country where most people don’t have a fraction of that opportunity.

    The problem with Harvard’s policies isn’t that it doesn’t give brilliant thinkers another leg up in life. It’s that it hurts Harvard, it hurts its students and it hurts society badly. Here I have to switch from talking about Harvard, due to lack of personal experience, to talking about my school, a member of the prestigious League of Universities that Play Football Against Harvard. I’ve TA’d two large classes now, and attended a number of teaching seminars, and one question that has taken up more time than any other is “how do we deal with large classes with significant differences in student ability?” And it’s a real problem. I have worked with professors desperately struggling to teach a Theory of Computation course where one third of the students are bored out of their minds and another third simply can’t get their heads around simple notions like decidability! At a Harvard Football League college!

    I can imagine that if Harvard simply accepted and enrolled the top 1000 students every year, Terence Tao would still be bored in class. But a few schools down, you would still have stellar students, but you wouldn’t have that kind of variance among students. And that means you can teach them according to their abilities, it means you can challenge them, it means you can throw away the James Stewart Introduction to Calculus, and teach rigorous analysis. And even down the line, everyone is better off in a world where you can challenge students, where students are taught according to their abilities, not the lowest common denominator. And if there is any advantage to a college education, that advantage would be immeasurably amplified in a world where we challenged every student – where we had the ability to do so.

  47. Scott Says:

    jk #35: OK, point taken. There might have been an extremely brief period in the 1910s and 1920s, after standardized testing was introduced and before the antisemitically-intended “holistic” policies, when the Ivies were using admissions criteria that I could more-or-less get behind. On reflection, though, by “returning extracurriculars to their rightful place,” I meant returning to the true meaning of the word, not to any specific historical period. 🙂

  48. Raoul Ohio Says:

    SO readers might be aware of Joel Spolsky, creator of Stack Exchange, Trello, etc. Like Scott, Joel blogs about all sorts of CS related topics: http://www.joelonsoftware.com/

    In relation to college entrance criteria; Joel has at times discussed how he selects new employees for Fog Creek Software. He gets top applicants by having a super cool place to work. On hiring decisions, he is impressed by straight A’s since forever. His theory is that this shows that you perform even on the courses that you are not interested in. Fair point. Probably a lot of SO readers (including me) are like Evan; a good test taker with spotty grades. Maybe we would have taken Joel’s advice?

    You may or may not agree with Joel on this or other points, but he is worth reading as he takes on a lot of topics and explains his reasoning, and is always entertaining. For any CS students out there, FCS has summer internships.

  49. Dave R Says:

    In the end, the employers will be the ones moving to standardized tests, because they can’t go by the college/college grades anymore (analogously to the way they can’t trust high school grades now) for all these ridiculous reasons.

    Which at the end of the day is the thing that makes the most sense, anyways.

  50. Scott Says:

    Dave R #49: But there’s a major factor you’re leaving out. Employers in the US are basically barred from using standardized tests (anything too similar to an IQ test), because of a 1971 Supreme Court decision. Indeed, some economists theorize that this is the reason why companies continue to place so much weight on which college applicants went to, despite all the issues with that metric: because it’s the easiest proxy for IQ that they can use while staying within the law.

    (On the other hand, I know that tech companies like Microsoft and Google are famous for asking puzzle questions in job interviews, like “why are manhole covers round?,” as well as Fermi questions, like “how many gas stations are there in the US?” Maybe that’s a situation just waiting for a Supreme Court challenge as to whether they’re administering IQ tests in disguise. 😉 )

  51. Scott Says:

    lewikee #44:

      I don’t see why private universities “should” change anything. They can make it their policy to only admit legacies that work full-time in soup kitchens for all I care. They answer only to themselves.

    No, I don’t think it’s that simple: in the US, even “private” universities receive lots of federal funding, and that funding is tied to their accepting many federal rules, about affirmative action, sexual harassment, and many other issues. So, speaking as an employee of a private university, I’d say that what we do is not just our business: it’s everyone’s. 🙂

  52. Scott Says:

    eitan bachmat #36:

      Following these experiences, what I really dont understand is why Scott, as a 15 year old, you even bothered with the American system, you should have come to Israel for a good, inexpensive and fun college experience, with essentially automatic acceptance.

    Thanks for your entertaining comment!

    Yes, maybe I could’ve moved to Israel; certainly Dana never had to deal with anything like the American admissions process (and has a hard time believing me when I try to explain it to her).

    But there’s one glaring problem: given the quality of my Hebrew, would the Israeli school officials have judged me to have an IQ of 90, as the American officials judged your research-mathematician son? 😉

  53. NKV Says:

    It would be nice to know if kids with high standardized scores do “better” when educated in an environment with all-rounder kids than in an environment with only kids with high standardized scores. IITs in India are an example of admission purely via standardized tests; that creates a very different culture than Harvard.

  54. Scott Says:

    Sam Tobin-Hochstadt #28:

      Pinker’s essay was good for the first two-thirds, but then it veers into genetic determinism and blithe ignorance about some of the consequences. Sadly, you end up endorsing the latter. Hopefully you don’t actually think that eliminating black, Latino, and working-class representation at Harvard and MIT would be a good thing.

    “Genetic determinism” is just a boo-phrase used to attack opponents without engaging their arguments. Absolutely no one thinks that genetics can explain all the variance in abilities between people (otherwise, how could there exist differently-abled identical twins?), whereas only someone ignorant of the data could maintain that it explains none of the variance.

    As I said in the post, I suspect that working-class applicants (especially from immigrant families) would do better under a test-centric system than under our current “eye-of-newt-wing-of-bat” system, which has evolved to become extremely tailored to the status games of the upper middle class.

    Also, I said explicitly in the post that I would have affirmative action for underrepresented minorities at my ideal university. I also support using affirmative action to even out the gender ratio, as Caltech, MIT, and other tech schools do today. What more do you want??

  55. Ben Standeven Says:

    @Darrell Burgan #40:

    Why would you want an evaluation method for those sorts of things? A university that only takes students who have demonstrated creative, philosophical, and ethical sophistication would be like a hospital that only takes patients who can present a clean bill of health.

  56. Darrell Burgan Says:

    Vadim #43:

    You have a great point. In the case of our model, we are able to control gaming of the model by restricting how often a person is assessed. Also, the “right” answer to any question is impossible to know, since it totally depends on the job they have, the company culture, and several others things. There literally is no right or wrong answer, only a statistical match, so it is almost impossible to game.

    Anyway, we’re already using standardized tests to assess merit, and people are already gaming them. That is status quo. My point is simply that the standardized instruments we’re using today are woefully insufficient and I see no reason the scientific method cannot be brought to bear to improve the situation.

  57. Darrell Burgan Says:

    Scott #50: you said

    Employers in the US are basically barred from using standardized tests (anything too similar to an IQ test), because of a 1971 Supreme Court decision.

    You’re right that companies cannot do IQ testing, but they are pushing that envelope right up to the limit. Companies are now assessing candidates in all manner of ways. Tests of “mental alertness and flexibility” are common. They may not say “IQ” on them, but they’re testing basic intelligence.

  58. William Hird Says:

    @Ben #55

    Then why not just pick names out of a hat, that way nobody can bitch about not getting in 🙂

  59. Scott Says:

    Incidentally, if there’s one thing that might cause me to reevaluate the views in my post, it’s not so much any counterargument that’s been offered, as the fact that Lubos enthusiastically supports me. 😉

  60. Dave R Says:

    Hi Scott – it is my understanding that employers can test for cognitive skills as long as they are relevant to the position under consideration.

    For example, Google can test potential software engineering hires for critical thinking/problem solving skills by asking a tough algorithms question or brain teaser, and Goldman Sachs can ask a tough calculus question to someone who wants to be a quant.

    On the other hand, Google can’t make the candidates for building manager answer these questions, as they would be mostly irrelevant. It’s my understanding the intent of the law is to avoid these types of tests being used to discriminate. For example, I believe the New Haven firefighters lawsuit claimed that asking difficult math questions on the entrance exam was discriminatory, as they were not directly relevant to being a firefighter and the results were racially disproportionate. I guess this type of thing used to happen a lot, for reasons you can imagine.

  61. Matt2977 Says:

    Scott #34 and #37:

    You write “you fail even to ask the key followup question: So how do you propose to measure those qualities in 17-year-olds? What’s your alternative evaluation method? Should admissions officers be trying to peer into the applicants’ souls, to look for ethical depth?”

    My answer? I don’t know! That’s the answer a humble person would give, considering the sorry state of all the metrics we currently have available to us and the staggering amount of uncertainty present in the system. I would never have written an essay in the New Republic declaring, as Pinker does, that the solution to the problem is simple and obvious. It’s not!

    (And speaking of humility, notice how Pinker tucks in that he was named “Harvard Yearbook Favorite Professor.” Come on — was that really necessary?)

    At best, schools have decided that what is needed is some mixture of standardized tests, courses taken, high school reputation, grades, class rank, extracurriculars, essays, recommendation letters. That is, pretty much what we do now, except for obviously dropping legacy and athlete students.

    And what’s the “correct” mixture?

    First we’d need to define “correct,” and then we’d have to grapple with the job of finding the balance to produce it. Those are hard questions, and simply declaring that testing is the “best” option is far from obvious. It’s the cliched search for your keys under the lamp post because that’s where the light is.

    My suggestion would be for people to come together to decide what they actually want, and then to do some serious research into ways of testing for it. But, frankly, any test one devises is going to run into the trouble you mention, namely, creating a “giant industry devoted to manufacturing the semblances” of those qualities.

    Right now, SATs are just a small part of the application. But if they ever become the primary part of the application, the industry devoted to SAT testing in America will only get worse. The stress of doing all those essays and getting high grades will be replaced by the stress of preparing for massively-high-stakes testing–and if you happen to have a bad week, then you’re screwed. Added to that, as was mentioned in the thread earlier, we already have schools that devote precious class time to preparing students for standardized testing, and it’s utterly ruining our school systems and making teachers and students miserable.

    And heaven forbid there’s a kid who’s a deep and brilliant but slow thinker!

    What I notice in both Pinker, Motl, and your responses here is that the admissions policies you all favor just so happen to tilt toward applicants like yourselves. An astonishing coincidence! To me, that’s always a reason to be skeptical. I’m a great test-taker too, but that makes me extra careful about assuming that the metrics that single me out are the “correct” ones.

    Now to your quotation about Feynman. Yes, Feynman was “human.” But the reason why that was so surprising to people was because of its rarity. Feynman was a huge outlier among people with his mathematical brilliance! If you give tests that single out people with Feynman’s intellectual talents, you’re not going to get a bunch of Feynmans. You’ll get a bunch of Diracs, and Wieners, and Pinkers.

    And that’s all well and good — those people are important! They are a crucial part of our intellectual elite, and we shouldn’t have a system to removes them from the system! But if those are primarily the people whom we’ll get filling the undergraduate populations of our top schools, then we’d be missing out on a huge other component.

    That component is hard to see if it doesn’t matter to you or you can’t perceive it. It’s telling when you write “In 7 years teaching at MIT, I have yet to notice any lack of humanity among the students here, let alone a “definite” lack.” And Pinker writes “I have spent my career interacting with these students, and do not recognize the targets of this purple invective.”

    Sure! Because, as you wrote, later, “social effects can easily run the other way: living at Telluride House at Cornell, I was surrounded by literary theorists, people liable to use words like “scientism,” “simplistic,” or “hegemonic” in a sentence. The result? I felt “something missing in an environment like that,” and one of the only other math/science people in the house became my lifelong friend.”

    And you’re right! There was something missing in an environment like that — b.s. detectors, logical and evidence-based thinking, humility, scientific analytical skills, etc. We need those things too! In your case, the lack of those qualities among literary types was distinct. But in the other direction, literary types or musicians or artists or philosophers might find other crucial human qualities to be lacking when surrounded by engineers.

    Heck, I’m a sciency type, and in college, I found the utter lack of interest in philosophical questions among sciency and engineery types to be profound and dispiriting.

    We need a whole host of human qualities in our top institutions of higher learning. The fact that you and Pinker don’t see some of those qualities isn’t evidence they don’t exist.

    If anything, it’s evidence that you both have blind spots, and that our educational institutions have failed in a huge part of their mission — namely, to expose you to other perspectives, to help you develop a capacity for empathy, to teach you that there is more to the world than is obvious to you, and to broaden your ideas of important human qualities beyond those you valued before you went to college or that characterize yourself and people who resemble you.

    It’s telling that Pinker’s list of things colleges should teach kids are missing out on that extremely important part of the mission of our educational institutions. Nor does his list include philosophy, which, by the way, Pinker has argued elsewhere is a waste of time because it’s somehow been made obsolete by science.

    You know where you can learn a little about “building a self” and other supposedly silly pursuits like that? In a really good philosophy seminar, of which there are unfortunately very few. Clearly Pinker has never taken one, or he’d know something about it.

    As for you, I’m immensely surprised! Perhaps you think that your capacity for philosophical depth and analysis are typical among those who might resemble you in their primary career interests or on standardized tests, but I think you sell yourself short. You, sir, are a huge outlier, and I frankly wish we knew how to test for that.

  62. Clayton Says:

    Oh boy – when I read the title of this post, I thought “academics” referred to the class of people, not the category of collegiate offerings, and I was indeed excited…

  63. Douglas Knight Says:

    Scott, you say that maybe employers use diplomas because 1971 decision bans IQ testing, but that very same decision banned using diplomas.

  64. Haelfix Says:

    Not to turn this about affirmative action, but i’d just assume we have those programs active in middle school and high school and not at the higher education level (At some point you do have to impose some sort of cull based on merit)

    Instituting them at the undergrad and graduate level leads to absurdities and high drop out rates whereas its the leaky pipeline that is the primary culprit for the underrepresentation of minorities etc.

  65. David Says:

    +Eitan Bachmat:
    I’m a fan of the Israeli system as much as anyone, but you neglected to mention the survival rate in the more demanding study programs with low acceptance standard. If memory serves the survival rate for first year math/physics/CS in HUJI in the early 90s was around 60%; as far as I know it hadn’t changed much, and French friends told me that survival rate there is similar.
    I actually think it’s a very good method (first years performance is a very strong predictor of academic success), but this arrangement is not a minor point.

  66. Scott Says:

    Matt #61:

      If you give tests that single out people with Feynman’s intellectual talents, you’re not going to get a bunch of Feynmans. You’ll get a bunch of Diracs, and Wieners, and Pinkers.

    With your image of a standardized test failing by picking out merely “a bunch of Diracs, and Wieners, and Pinkers,” one can clearly see several places where you and I part ways.

    Most obviously, that’s like, the most awesomest test ever created. You don’t want it? Dude, give it to me!! 😀

    But I think you ironically overrate standardized tests: there’s no standardized test to identify the profound mathematical creativity of a Dirac or a Wiener. (For now, let’s set aside Pinker, whose gifts are in a different direction.)

    At best, there are tests of mathematical skill, which (in place of anything better) can help to create the sort of baseline environment where a Dirac or a Wiener can thrive—as, in fact, they did. There are also tests of verbal skill, which, while failing to identify the truly great writers and philosophers, can and do help to create the baseline environments where those people can thrive.

    Speaking of which, need I remind you that math is only 1/3 of the current SAT? There are plenty of math nerds who get killed by the verbal portion, while those who spent their childhoods reading literature fly through. Personally, though, rather than simply summing all the scores, I would make each portion much harder, and then take the maximum score. I want the students who ace the math portion even if they have the reading skills of a third grader, and—please read to the end of the sentence—I equally want the students who ace the verbal portion even if they can’t solve 2x+5=11. A well-rounded student body is great, but there’s no need to construct it out of well-rounded students.

    Finally, to return to Pinker: for chrissakes, he’s a bestselling author, with award-winning works that range over psychology, anthropology, linguistics, history, and philosophy (with just tiny sprinklings of math). He’s also a superb literary stylist and speaker. If he’s your idea of a narrow-minded math nerd, then may we all strive to be equally narrow.

  67. Scott Says:

    Michael #23 and Francis #29: To clarify, I don’t consider myself to have been “intellectually deprived” in any way, shape, or form by having gone to Cornell (!)—quite the contrary. For starters, Cornell’s CS department is ranked 6th in the US, well above those of (e.g.) Harvard and Yale. I got an excellent grounding in CS and math at Cornell, learning from brilliant professors like Bart Selman and Jon Kleinberg. My classmates included Ryan Williams, who’s now one of the greatest complexity theorists alive, as well as David Liben-Nowell and Pedro Felzenszwalb, now also CS professors—not exactly a bunch of slackers. And importantly for my future career, I got in from Cornell to my #1 choice of graduate program, which was Berkeley.

    So my point here wasn’t to ask for anyone’s pity (!): I’m now very much where I want to be in life, and it’s far from obvious that doing my undergrad at MIT or Princeton rather than Cornell would’ve improved the outcome in any way. (Maybe it would’ve been better, maybe worse: I have no idea.) Would that everyone else could have a tenth the opportunities I did.

    My point, rather, was that there seems to be something obviously screwy about an undergrad admissions process that, not just once, but on a fairly regular basis, rejects the very same students who later get recruited to the institution as its faculty—even though the faculty pipeline is at least 20 times narrower than the undergrad one, and even though the same qualities that made those people attractive as faculty were usually in evidence much earlier. I came out of that process just fine; others might not be so lucky.

  68. Scott Says:

    Incidentally, Matt #21:

      But could you imagine selecting PhD students just based on standardized tests?

    In my experience, PhD admissions are fundamentally different from undergrad admissions, for the following three reasons.

    First, PhD students are recruited, not (just) to develop selves or take in the sum total of human knowledge, but for a much clearer purpose: namely, to do research in their chosen field. For that reason, PhD admissions are in many ways closer to job interviews than they are to undergrad admissions. The focus is on a relatively well-defined question—can this applicant do the work?—so there’s much less scope for the kind of chicanery that standardized tests are useful for counteracting.

    Second, most PhD applicants these days have at least some experience doing research (or trying to do research) as undergrads. For that reason, one has the opportunity to judge them much like a prospective employer would: by how well they’ve already done at the task they’re being hired to do. That sort of evaluation, when available, is even better than standardized tests.

    Third, and probably most important of all, the people making PhD admissions decisions are the same professors who students will be working with if they enroll. So those professors have an enormous incentive to try to hire based on competence rather than political criteria. Again, this means that standardized testing is less needed as a corrective.

  69. N. Says:

    In France, admission to the grandes ecoles is governed by admissions exams that are aimed to discriminate at the very top end. This has resulted in a set of (probably) equally perverse incentives for high schoolers who must spend a couple of years after high school in paid cram schools trying to pass these exams if they want to be admitted. This would seem to be more scholarly, but I don’t think it actually results in a fairer result or better outcomes for the graduates, schools, or the country.

    To be honest, I think that the US system *taken as a whole* is pretty good. If you want a basic cutoff for decent SAT score and grades, apply to a public school. If you want to be evaluated on your potential to be a future member of the ruling class, aim for the ivy league. If you’re a math prodigy with no other redeeming qualities, you can probably convince one of MIT, Caltech, CMU, Harvey Mudd, etc. to accept you (as you did). If you completely and utterly screwed up in high school, you can *still* go to a junior college, transfer to a four-year university based on grades, and gain admission to a graduate program on your own merits.

    The real injustice comes when you tell people that the only place that really smart people belong or that you can get a good education is Harvard or MIT, and that you must pay $50,000 a year to attend a school or you’ll never get ahead in life, and underfund or dismiss the public universities that are the great equalizers here.

  70. Scott Says:

    N. #69: Thanks for your comment; there’s plenty in it that I agree with.

    In particular, I know from personal experience that it’s profoundly false that “the only place you can get a good education is Harvard or MIT”—I said as much above, and Pinker also said it. Even if I were 20 times smarter than I was, I would’ve found courses at Cornell that were too hard for me. Indeed, I was offered a nice scholarship at Clarkson University (where I spent my senior year of high school at The Clarkson School), and I believe I would’ve gotten a good education even if I’d chosen to stay there.

    In short, as soon as I was in the hands of America’s universities (and out of the hands of its high schools 🙂 ), I was never deprived for a microsecond of intellectual stimulation. I have only myself to blame for whatever I failed to learn from that point onward.

    My point was a more societal one: if we’re going to have elite universities whose real purpose is to groom (as you put it) “future members of the ruling class,” then why should those same universities also get to arrogate to themselves the veneer of intellectual excellence? Unless, of course, we were serious about drawing our ruling class from among the intellectually excellent. But in that case, Yale should probably be admitting the people with the highest SAT scores, rather than the George W. Bushes.

  71. Jeremy Says:

    As a recent MIT math-focused undergrad (and current Harvard math grad student) I see first-hand that MIT has rather different admissions criteria than Harvard.

    Harvard rejects some of the best high-school mathematicians. Recently they rejected a student who went to MIT and as a freshman immediately solved a grad student’s thesis problem in an advanced topic. This student also had good grades and performed amazingly on Olympiads. Harvard is simply not interested in selecting purely for academic talent or interest in their undergraduate population. I have known Harvard to reject numerous US IMO gold medalists.

    MIT, on the other hand, operates surprisingly close to how you want it to. They have spots on their application asking for AIME scores, and of the 100 or so USAMO participants I know, 95% were admitted to MIT. Basically, as a US citizen performing decently on the math Olympiads is a ticket to MIT.

    The only difference between the Olympiads and the SAT is that more high school students know about the SAT, and that is of course a serious difference.

    Your perspective, however, is a little close to your own background. I know math faculty at MIT who rail against the Olympiads, and declare that they should not be a factor in admission. These tend to be the types that are less competitive and more collaborative, or only got interested in math later in life. The Olympiads represent excessive focus on technique in a few relatively accessible areas; they actively discourage students from learning more advanced mathematics. They are exactly what comes of “gaming the system” where standardized tests represent future opportunities.

    Anyway, I think there are clear disadvantages to all systems, but I actually think that MIT (as opposed to Harvard) strikes a reasonable balance.

  72. tas Says:

    Could there be a more scientific approach? Is there research on what are the best predictors of success in college and later life?

    I would prefer a system that didn’t place so much weight on elite universities and highly selective admissions.

  73. Will Says:

    Scott, what do you think of the idea that selective private universities (and maybe a few of the most selective public ones) should set some threshold for qualification, and then among all qualified applicants select who to admit by lottery? This has some of the advantages of your proposal, but avoids some of the horrors of the all-important standardized test that people above bring up.

  74. J Says:

    Please take a take on this http://www.kpopstarz.com/articles/111389/20140913/fluid-dynamics-quantum-mechanics.htm
    please.

  75. Rand Says:

    It’s interesting to note that MIT Computer Science is the only school (to my knowledge) to say on its PhD admissions form “Do not send us GRE scores. We will throw them out.” (Paraphrased)

    That’s partly because the (old) math section was only capable of filtering out students who were terribly confused about what they were applying to, and the CS Subject test was a travesty. (The general GRE verbal section actually did an okay job of sorting students at the high ends.)

    But I’m wondering: Scott, if we had a good test for high mathematical ability would you take it into account when taking new students?

    Even if I were 20 times smarter than I was, I would’ve found courses at Cornell that were too hard for me.

    Funny, what course were Professors Wittgenstein and Von Neumann co-teaching? (You know, some 40 to 50 years after their deaths.) Really, I’d like some examples on such courses, as well as some detail on how we’re scaling your intelligence here.

  76. Darrell Burgan Says:

    Ben #40, you said:

    Why would you want an evaluation method for those sorts of things?

    I”m not advocating a standardized test to evaluate someone’s sense of fashion style or ability to summon clever witticisms. I believe strongly that universities should be about teaching and research, not about sports or sororities or outrageous drinking.

    My point is that today’s standardized tests for measuring intellectual bandwidth, for finding the candidates who truly will excel and move humanity’s understanding of the world, are myopic. Even within mathematics, they only test a vast subset of the subject, and I would submit they miss the fundamentals.

    For example, do you really think the SAT could find a student who has a stunning ability to visualize complex objects but tests poorly? What about the student who cannot see but can hold large systems of partial differential equations in their head? How many of science’s greats would have done well on the SAT?

    And why is math and science the only academic study worthy of pursuit? We can chuckle about people who have PhDs in classical greek architecture, I suppose, but are you really suggesting that there is no intellectual value in hard anthropology? Do you really suggest there is no possibility for hardening sociology into a truly predictive practice? What about political science – do you really suggest that there is no value to people who have gone deep on policymaking and governance?

    That’s my point. We certainly want candidates to be judged based solely on merit. Our model of merit, however, is flawed and needs improvement. I’m confident that science can meet this challenge.

  77. gasarch Says:

    A few more random thoughts:

    The process for Grad school works pretty well. Maybe we could mimic that.
    a) Grad students know what they want- people who can do research. That very focus (as opposed to “building a community of scholars”) helps. So first colleges need to decide what they want.
    b) As a result, grad school don’t care about extracurrs and care more about GRE’s and Grades (in certain courses) and of course research. Is their an analog of research for ugrads— there could be.
    c) Scott mentioned this briefly– the profs make the grad admissions decisions. if the profs made the undergrad decisions (perhaps by major) then that alone would change things as they have to live with the consequences of their decisions. If the math folks are happy with someone whose Math SAT is 3(Verbal SAT)/2, fine. And of course this may lead to harder exams and more discernment.

    2) FYI- I went to SUNY Stonybrook undergrad and was Fine- I took several Grad Math courses (including Joel Spencer’s course on Combinatorics which included Ramsey Theory!) and one CS Grad course (Theory with Harold Gelertner, who does AI normally). Then Harvard for Grad School which was a healthy but rude awakening.

  78. Scott Says:

    Will #73: I don’t like the lottery system, because I don’t agree with the premise that everyone is basically indistinguishable once they pass some threshold of being able to do the work. Indeed, given the numbers we’re talking about (maybe a dozen universities, each with ~12% or lower acceptance rates), a lottery system would run a significant risk that the best student the world has ever seen would be shut out of every top university in the country, just by chance.

  79. Raoul Ohio Says:

    One problem with any standardized exam is gaming the system, outright cheating, ringers taking the test, etc. Although technology can help lessen the impact, the ingenuity of scammers knows no bounds.

    Here is an anecdote to illuminate the situation.

    In the 1970’s or maybe 80’s, there was a lot of press given to a huge cult in the US run by a 15 year old Guru from India, who appeared to be an adult, and was married to a 25 or so year old from the US. It was the top of the “alternative lifestyle” world for a couple of years. (Anyone recall the name, so I can Google for an update?)

    Anyway, I asked a prof from India what he thought of this guy. He replied that anyone from India can tell you what the deal is: In India, lots of things are determined by standardized tests given at certain ages. It is not uncommon for families to not register births for a year or so, so that their kids can take the 10 year old test when they are 12, etc. In extreme cases, 10 or 15 years gets lopped off the age of kids. The guru is really 25 or 30, but his official age is 15. This apparently impress’s the people in his cult.

    Plenty of similar stuff all over. In the US, schools in poor neighborhoods are financially punished if their kids do not do well on standardized tests. Anyone want to guess what winds up happening?

    My guess is that there is no simple solution.

  80. Scott Says:

    Rand #75:

      Funny, what course were Professors Wittgenstein and Von Neumann co-teaching? (You know, some 40 to 50 years after their deaths.) Really, I’d like some examples on such courses, as well as some detail on how we’re scaling your intelligence here.

    OK, I was engaging in hyperbole—I don’t even know what it means to be “20 times smarter.”

    What I should have said was: at the time I graduated Cornell, I wasn’t anywhere close to exhausting the academic resources there that would’ve interested me. There were a dozen or more interesting CS courses that I hadn’t yet taken, I hadn’t taken any graduate math courses, and I would’ve been happy to take more courses in physics, philosophy, and creative writing.

    (On the other hand, I was already getting interested in quantum computing, and at that time, there was no one at Cornell who worked on it—David Mermin hadn’t yet gotten interested. That, combined with my desire to see a new place and the general discouragement of ‘incest,’ are why I didn’t apply to stay at Cornell for grad school.)

  81. Scott Says:

    J #74: Pilot-wave theory is at least a 60-year-old idea. To summarize the situation in a blog comment:

    (a) You can reproduce all the successful predictions of QM using pilot waves—indeed, get something that’s experimentally indistinguishable from QM—but only if you introduce a “guiding potential” that contains all the complexity of the ordinary quantum-mechanical wavefunction. So, this sort of move does nothing whatsoever to cut down on the exponential vastness of Hilbert space: the only thing it buys you, is that now you can say that in addition to the wavefunction, there are also “real, actual positions” for the particles.

    However, those real, actual positions aren’t directly measurable; if you try to measure them, you’ll get exactly the same probabilistic predictions that you’d get from ordinary QM. For that reason—as well as more technical issues, like the non-uniqueness of the evolution rule for the actual positions, and the failure of the actual positions to play nice with relativity—most physicists regard the actual positions as metaphysical baggage they can do without, if occasionally helpful for visualization.

    (b) If you want to get rid of the vast, exponential wavefunction, and say that quantum mechanics can be completely understood in terms of classical fluid dynamics, that actually works quite nicely—as long as you’ve only got one particle!! As soon as there are even two particles, your fluid model will fail to capture effects arising from entanglement. And this is not some technical issue that could be gotten around with more research: the whole point of Bell’s Theorem was to show that no classical local-realistic theory will get entangled particles right. (Though even today, there are people determined not to understand this…)

  82. Highly Adequate Says:

    One thing’s obviously missing from your understanding of how a university like Harvard might select for intellectual achievement: any appreciation of any discipline outside of STEM.

    Harvard doesn’t want to select just the promising STEM students. It want to select, as well, the most promising intellectual leaders in all disciplines, including biology, medicine, the life sciences more generally, the social sciences, the humanities, and writing.

    There’s no particularly good reason to believe that the most promising student in many of these categories will have nearly perfect SAT scores — indeed, even the most promising STEM students may not have nearly perfect verbal scores.

    I should think it pretty obvious that overlooking imperfect SAT scores and paying attention to other factors is the way to go for Harvard’s purpose.

    Of course it’s just one example, but consider William Gibson, the famous seminal figure of Cyberpunk sci-fi. According to Wikipedia,

    “[Gibson] took the SAT (Scholastic Aptitude Test) exams, scoring 5 out of 150 in mathematics and 148 out of 150 in the written section, to the consternation of his teachers.”

    This should suggest the gap that may exist between verbal talent and technical talents. (Not that Gibson could have gotten into Harvard — if anything standardized testing, and a bunch of other factors, would have stood in his way.)

    If one looks at Harvard’s overall record in selecting students who go on to great intellectual achievements across the board, it’s pretty hard to fault its criteria, whatever they may be.

    Ask yourself: which university does a better job across the board?

    And, of course, the further point remains that Harvard really does seek out the future leaders of America, and not just intellectual heavyweights. There too, it’s hard to fault it, given its success in the past.

  83. Will Says:

    Scott #78:

    I don’t like the lottery system, because I don’t agree with the premise that everyone is basically indistinguishable once they pass some threshold of being able to do the work.

    Well a lot of people in this thread don’t like your system because they don’t agree with the premise that standardized tests are a perfect way to measure academic potential and that 16- and 17-year-olds should devote their lives to test preparation. But wait… that’s not the premise of your argument. Your argument is that your system just might be the least bad system, despite the fact that standardized tests aren’t a perfect measure of academic potential. Well, then…

    a lottery system would run a significant risk that the best student the world has ever seen would be shut out of every top university in the country

    Of course, some minority of spots could be reserved for exceptional students but the majority would go to lottery winners (similar to your system).

  84. Tyler Says:

    Many good points being made. But I think the core of the discussion is off-base, on 2 levels:

    1) How can we debate the best way for admissions to achieve their goals if there is not even agreement on what those goals are?

    2) Does it even make sense to talk about what those goals are, when it really is up to each and every university to decide that for themselves?
    ————-

    1) The above comments have so many suggestions for how to improve admissions to get the right type or distribution of students, yet isn’t a necessary prerequisite for these suggestions the establishment of what the right type or distribution of students is? The title of the post itself implies that such an establishment does not exist, or at least is not set in stone and should be changed. You can’t productively talk about a maximization problem when there is any discrepancy about what objective function to maximize.

    2) Additionally, I would argue that it is not necessarily possible to eliminate discrepancy between objective functions. This whole discussion seems to be under the context that there *should* be some standard process that students go through in order to apply to any of the many options. But what if School 1 WANTS to focus on athletics, and School 2 WANTS to focus on CS, and School X WANTS to focus on some arbitrary combination of all the things they might care about. Why should there be some single test or process or whatever, that is a blanket solution for all the possible needs of a school’s admissions process?

    The main counter-argument I see against this admissions process customization (which already occurs for some schools defining their own extra processes, but not to that large a scale) is scalability. Students/parents want to have a vast array of options by taking as few tests and such as possible. But if having such a blanket solution requires a system with questionable integrity, the sacrifice just doesn’t seem worth it.

  85. Will Says:

    I think maybe one thing that’s missing here is a discussion of the purpose of undergraduate education in America. For people like Scott and many of the commenters here (including myself), the purpose of college was roughly to follow our intellectual curiosity and at the same time to prepare ourselves for further study in an academic field. For people in engineering school or on the pre-med track, the purpose was to prepare for a certain career (not to deny that these people have intellectual curiosity also). But most students don’t fall into these two categories. The most popular careers among Harvard grads are in finance and consulting.

    So Scott seems to be working from the assumption that the very best students should be in the very best universities. And that certainly makes sense if the purpose of college is to train academics and researchers. A talented researcher will thrive most around talented mentors and a thriving, talented researcher is beneficial to society (or so we like to believe). But if Harvard grads are just running off to Wall Street, who really cares if they had a 1300 or a 1600 on the SAT?

    And what if we radically propose that the purpose of undergraduate education is to educate undergraduates?

    I’m not entirely sure of the point I’m trying to make. Maybe the entire concept of a prestigious college (college as in undergrad program) is toxic, but it’s the price we have to pay the beneficial(?) concept of the prestigious university.

  86. Scott Says:

    Will #83: OK, let me try again. It seems to me that we can clearly, easily do much better than the lottery system. Whatever test you use to set the threshold for the people entered into the lottery, that test presumably contains (or could be written to contain) a lot of highly-relevant information about the gradations in ability of the people who are over the threshold. So why throw away that information?

  87. Will Says:

    Scott #83: Why throw away that information? Sorry if I wasn’t clear about this. The big advantage over your system is that it would mostly avoid the test-prep arms race (see comments #69, #79) which sounds truly awful and is an actual thing happening in other countries and not just a hypothetical. So that ambitious 17-year-olds could spend their time following their interests instead of studying all day trying to squeeze out 10 extra points on the entrance exam (or instead of having to go thru the absurd rituals that you describe in the current system).

  88. Scott Says:

    Highly Adequate #82: (sigh) See my comment #66, where I explained how at my ideal university, there would be extremely hard entrance exams in a variety of areas, and we’d seek out the students who had done spectacularly well on one of the exams, even if they’d bombed the others: for example, if they’d aced the verbal test, even if they’d bombed the math test. (Or at least, we’d combine the scores using something like the 4-norm, to give disproportionate weight to “outlier” high scores.)

    So yes, I do have more than STEM in mind here. Personally, I remember liking the SAT verbal section a lot—especially the reading comprehension part. As I thought of all the students who were going to be marked down for leaping to conclusions about what the author of the passage intended, and not reading to the end to find out, I felt like there was some tiny measure of justice in the world. 🙂

  89. Darrell Burgan Says:

    Tyler #84:

    I think you may have the essence. There are varying goals for admissions, and universities have varying overall missions. Harvard may not pump out as many high-end mathematicians or scientists, but their business and political alumni are rather peerless. Different goals, different admissions practices.

  90. Scott Says:

    Tyler #84 and Darrell #89: Sure, by all means, we should have different admissions standards for different universities! As several commenters pointed out, this has been important historically: for example, in the era when Harvard, Princeton, and Yale effectively had Jew-quotas, many Jewish students were still able to get in to CCNY and U. Chicago, which prioritized academics more. And today, it’s a very good thing that Caltech (for example) doesn’t have the same criteria as Harvard.

    Having said that:

    (1) As far as I know, there are zero top-tier universities in the US right now whose admissions policies would pass what I’ll call the “professor test.” Namely: if the professors in some particular department, on being shown an application, would say, “holy crap, admit this student!!,” then the student would reliably be admitted. Caltech (where professors are heavily involved in reading undergrad applications) might come the closest, with other tech schools not far behind. But all of them put a lot of weight on high-school grades, a metric that in some cases reveals more about the school or the teacher than about the student.

    (2) One could, of course, start one’s own university, with more-or-less whatever admissions criteria one wanted (consistent with federal and state law). The problem is that the reputations of the top universities usually only change over a timescale of centuries, if that. rrtucci #3 notwithstanding, I expect that Harvard will still be Harvard and Princeton will still be Princeton until the collapse of civilization. So if one feels that the top universities’ admissions standards are screwy, in practice one doesn’t have the option of simply founding a new university (or beefing up an existing one) to outcompete and out-prestige the big players, the way one could with an inferior restaurant or web browser. At most, one can do that sort of thing with individual departments. In practice, I think, one’s only real option is to try to persuade the existing top universities to change their standards. That’s why I’m delighted that someone of Pinker’s eminence has thrown his weight behind this.

  91. Highly Adequate Says:

    Scott #88:

    Well, I did read your original post and a good number of comments down. No, I didn’t get to #66, where, I assume from you said, you first really acknowledge that there are other intellectual areas Harvard might have a legitimate interest in other than STEM. I certainly saw essentially nothing in your original post acknowledging this, apart from some concession to “musical and artistic” talent among a number of other items.

    I think it’s pretty obvious that selecting the best students in STEM is a lot easier to be done by standardized testing than in other intellectual realms. In a STEM class, even at MIT, it’s pretty easy to know very quickly just how good a student is, for those who participate in class, and rank them down the line. This is far less true in most humanities classes at, say, Harvard. No doubt a high verbal standardized test is important in many humanities and social sciences, but it’s much harder to rank order who the best students are with such tests at the highest levels. Certainly the predictive power of the verbal GRE, even though it has a very high ceiling, is, while of some value, is not impressively high when it comes to predicting future research (around .2, as I recollect).

    Which brings me back again to the point I was making: that Harvard does remarkably well in choosing students who will become among the highest achievers across disciplines. I don’t know any other institution that does half as well — and neither Caltech nor MIT count, because of their narrow focus.

  92. Sam Hopkins Says:

    Scott, is it fair to say that you think that the admissions process for graduate schools does a better job (perhaps precisely because it likely to pass the “professor test” in light of the oversight of a real professor in the department)?

  93. Michael Dixon Says:

    Will and Tyler bring up what I was going to mention myself: these universities haven’t adequately defined their current purpose or mission (not just within admissions). I want to add that this isn’t an abstract or theoretical concern, I have directly encountered it many times. In my experience, it ended up irreversibly poisoning part of my undergrad education. I’ve restrained myself from posting this personal anecdote in the past, but perhaps it might be useful to someone here and to the discussion.

    I previously asked faculty at my school’s CS department what the philosophy behind their undergraduate program was. My interests at the time weren’t compatible with the templated undergrad education they offered. I took the time to genuinely see if my expectations should change in accordance to their plan or at least comprehend their policies. I darted around campus over the course of 2+ years meeting with dept heads, academic advisers, professors from various disciplines, and even the deans. Not a single individual could provide even a vague explanation for the current educational designs and goals. I familiarized myself with that specific puzzled look and the delightfully lost train of thought accompanying it. My concern wasn’t treated as a pressing problem. It was viewed as an amusing question that needn’t be answered. I was directed to individuals dispersed over multiple campuses. “Person X, they might know more about it” was what I was repeatedly told. As time went by, the people who might have understood the university’s purpose moved on. New, inexperienced appointees filled their place that had no better understanding than the others. My adventure had turned from a quest into an autopsy. Published documents, statements, and testimonials also evaded the question. I ended up choosing to challenge myself and pursue my own interests despite how it prolongs graduation.

    I believed that this was an isolated problem to this type of giant public school shrouded in bureaucracy. Certainly other universities, especially those who held universal acclaim, did not have the same issues? Events afterwards called that into question. I stopped by MIT Admissions office on a whim while I was visiting during a summer. I hadn’t planned on it, but I figured “why not satisfy my curiosity a little bit?” and “let’s get something to compare my experience to”. I asked them what their vision was when they select incoming classes. What was MIT hoping to see in these students and why do they think their environment supports that? What is their purpose and goal for doing what they do? I selfishly asked myself, “what was it that I was missing out on?”. The admissions officer I met with seemingly had no clue, but I cannot know for sure. They immediately gave up after realizing they didn’t have a real answer ready. Instead, they introduced me to a current MIT student that worked in the office. She and I had fun chatting for a bit. I got to learn about her interest in rocks and got to see her enthusiasm for the subject. Based on her experiences and her view of MIT life and academics, I couldn’t get a sense of the purpose, promise of fulfillment, or direction that MIT supposedly might have provided. So I unexpectedly left with the same uneasiness that I had when at my home university.

    I don’t know why exactly, but this crushed my spirits quite a bit. I was either denied the answer all this time or it simply doesn’t exist. I did everything I could to have the right attitude and accept things, but I became unconsciously stubborn over it. My brain refused to let me read or write, which impeded my ability to perform academically at all. It even barred me from completing work that I was adamant on completing. Since then, I’ve accepted the fact that I will not be able to reconcile my problem with undergraduate education while I am in it. Don’t get me wrong, there were plenty of awesome opportunities and I met excellent people (especially some supportive professors). I don’t excessively blame these schools for not knowing since they are providing something I’d otherwise not have. I don’t want to come off as if I feel I am entitled to a fulfilling education that is done my way. It might have helped if I understood the “why”s of their education. If they can’t justify their education, why should I have the least bit of confidence in it? At this point it is more likely that I’ll move on in life without finishing a degree. Perhaps one day, when they fix these issues, I’ll return.

  94. bhadra Says:

    Yes please. I much prefer the system in the uk where you are judged on your academics mostly. As for the indian system-it is incredibly stressful-far roo stressful, but poor kids do get into government colleges and once they do that they are pretty much set for life.

  95. Parent of young math whiz. Says:

    (Seeking advice:) What are the college admissions prospects for a kid who is very good at math, but less stellar in verbal areas, and no extracurricular brownie points.

    For which N and X does being in the top N (as measured by math competitions) give one a good chance of being admitted to institution X?

    What institutions should one be looking at if N=1000? What if N=100? And which institutions should a monolithically mathy kid want to got to?

    (For context, our kids are US born, but us parents are not US born nor US educated, and the US college admissions system is completely alien to us. I can’t fathom the importance of soup kitchen voluntourism.)

  96. jb Says:

    Further to Tyler #84: The US admissions process is pretty close to a free market. If adopting Scott’s approach (or any other) would further the interests of University X, then it’s pretty likely University X would have done it already. It’s not like something similar to the Scott/Pinker approach never occurred to them. Brown could admit a few more smart immigrants and a few less do-gooders from Manhattan, and it would slowly overtake the likes of Princeton—with respect to the SAT metric. But it doesn’t. Why? Perhaps the answer is that Brown (or any other university) only cares so much about what the SAT metric measures and that their current approach optimizes for what they really do care about.

    So it seems to me that Scott/Pinker thesis then seems to boil down to… damn it, Americans, why don’t you care more about the academic values that I care about?

  97. J Says:

    scott 81 Thank you very much. On your “Though even today, there are people determined not to understand this”…. why does the work is described as brilliant in the end? here is the mtit prof’s work? http://math.mit.edu/~bush/?page_id=484 and his student’s work http://math.mit.edu/~auoza/ and A video is here http://www.youtube.com/watch?v=nmC0ygr08tE.

  98. Darrell Burgan Says:

    Scott #88:

    … the “professor test.”

    Of course, the obvious problem is that no university could possibly tie up its professorial staff with endless reviews of incoming candidate students. So the goal has to be some kind of admissions program that approaches this ideal as closely as possible, without requiring the top minds to do all the evaluation. Hence the problem.

    Honestly, it’s the same problem in industry. When I consider a candidate for my team, the best approach is always to put all my best folks in a room and grill the blazes out of the candidate. We’ll definitely emerge with a good sense of whether the candidate will prosper or not. But we can’t always do that, because the best folks are just too busy.

    Interestingly, in my industry we also have a semi-standardized regimen for evaluating talent: they’re called “certifications”. The idea is that one can assume that people who have a given certification have at least a certain minimum knowledge of the subject matter. The problem is that such certifications are subject to the same amount of gaming as any other standardized testing. As such, I tend to not be particularly impressed when a candidate comes in with a certification. It doesn’t hurt them, but it doesn’t help them much. I want to know what they truly know how to build, and I want to know whether they’ll wither in a firefight. Call it “peering into their soul” if you want, but that’s what I have to assess. 🙂

  99. Jon Says:

    Scott #90

    You write ‘As far as I know, there are zero top-tier universities in the US right now whose admissions policies would pass what I’ll call the “professor test.” Namely: if the professors in some particular department, on being shown an application, would say, “holy crap, admit this student!!,” then the student would reliably be admitted. Caltech (where professors are heavily involved in reading undergrad applications) might come the closest, with other tech schools not far behind.’

    Three years ago I took my home schooled math kid to Caltech to talk to them. Even then, at 15, he would have been taking graduate math classes at Caltech. However he’d never done any English or history or science classes. So very pointy as they say.

    When I asked a math professor there whether my kid could get into Caltech his response was to suggest that he stop studying math and start jumping through the hoops that the admissions people insisted upon. He said that the professors had no influence whatsoever, and in fact had recently spent two years trying to get some amazing math kid in to no avail because admissions had refused. And this was a professor who sat on the admissions committee as an advisor, so he ought to have known.

    My math kid is now 18 and finally going off to college. Where? Not an American school. He’ll be at Cambridge where what they care about is your love for and understanding of your subject, not your extra-curriculars and performance on irrelevant tests.

    The only American school he applied to was Berkeley, which turned him down despite support from the math department there (so the evidence is that they don’t pass the professor test either). That seemed the likely result at any other top school in the US, so why bother applying?

  100. yme Says:

    Will #87: Ambitious 17-year-olds would surely be happy to be able to spend their time following their interests (rather than preparing for a test), which they would feel free to do under a lottery system, but they would just as surely be unhappy about probably not getting into a top school and being unable to do anything about it. I mean, if they didn’t care about getting into a top school, they could already follow their interests now.

    It seems to me that the answer is not to get rid of tests, but rather to test for whatever it is that you’re truly interested in. Then, it’s good if people prepare for the test: they’ll be learning exactly what they ought to be learning.

  101. yme Says:

    Darrell Burgan #76 asked: “How many of science’s greats would have done well on the SAT?”

    I’d guess, a very large fraction of them.

    It sounds like you think otherwise. Why do you think that?

  102. Scott Says:

    jb #96:

      So it seems to me that Scott/Pinker thesis then seems to boil down to… damn it, Americans, why don’t you care more about the academic values that I care about?

    Yes, 100% guilty as charged! Pinker and I aren’t trying to tell the elite universities how better to optimize their admissions processes around their existing values; instead, we’re telling them what they ought to value.

    In our defense, if (as many commenters have said) Harvard and Princeton are private institutions that can do more-or-less whatever they want, then surely Pinker can gripe about it however he wants in his article and I can do likewise on my blog. 🙂

  103. Scott Says:

    Sam Hopkins #92:

      Scott, is it fair to say that you think that the admissions process for graduate schools does a better job (perhaps precisely because it likely to pass the “professor test” in light of the oversight of a real professor in the department)?

    It’s not only fair; it’s a superb statement of what I think. I would love to see undergrad admissions become more like graduate admissions in its central guiding values (though of course, even in an ideal world they shouldn’t be the same, because of their different immediate purposes).

  104. Scott Says:

    J. #97: I haven’t talked to Prof. Bush (I should), but I have to believe that he understands Bell’s Theorem, and why it shows that these hydrodynamic models of quantum mechanics aren’t going to work beyond the single-particle case. I suppose he’s interested in pushing the analogy as far as he can, even in the teeth of that known reason why the analogy has to break down. More power to him, I guess! 🙂

  105. Scott Says:

    Jon #99: Thank you so much for sharing your story. It’s precisely because of cases like your son’s that I spend time on this issue.

    I’m sorry to hear that neither Caltech nor Berkeley would accept your son, despite support from the math departments—that both of those institutions failed the “professor test.” (I had heard that Caltech has heavy faculty involvement in admissions, but at least from your story, it sounds like it’s largely in vain.)

    I’m thrilled to hear your son was accepted to Cambridge, and I hope he does great there.

    Having said that, my advice to anyone else reading who might be in your son’s situation is: if you’re interested in attending college in the US, apply broadly. Just like with asking someone on a date, don’t “pre-reject” yourself; let them reject you if they aren’t interested. Even if Caltech and Berkeley (or Harvard, or Stanford, or MIT) say no, there are many other schools with excellent intellectual traditions that might give you a chance: Cornell, CMU, U. of Chicago, UT Austin… indeed, the process is sufficiently noisy that at least one of these places might let you in “just by accident.” (Having said that, going abroad is certainly another option, and I’ve heard that this Cambridge University is a real up-and-comer on the academic scene. 😉 )

    I can tell you that, in my seven years at MIT, one of my greatest professional satisfactions has been helping a couple of truly spectacular and unusual high-school students get admitted here. (Or at least, I think I helped: the admissions office gratefully acknowledged my letters of support, and the students got in, but I don’t know what role the letters played.) In your son’s case, you did exactly the right thing by getting the math faculty involved, but the next step (which didn’t happen) would’ve been for the admissions office to listen to the math faculty.

    Incidentally, this highlights a crucial point that I should’ve discussed in the original post. Many commenters here have tended to treat universities as monolithic entities—as in, “Harvard doesn’t have to prioritize pure intellectual achievement if Harvard doesn’t want to.” But of course, Harvard is composed of many factions, and what its admissions officers or its administrators want might be very different from what its faculty wants. (It’s certainly different from what Steven Pinker, one of its most famous faculty members, wants.)

    So I view Pinker’s article as partly a call-to-arms to the faculty of top universities: to take charge and impose on their institutions the high intellectual standards that come naturally to them, even if some admissions officers or administrators have to be dragged along kicking and screaming.

  106. Sam Hopkins Says:

    Do you think students should be required to take a wide range of classes? Part of the liberal arts model as I understand it is that you’re supposed to learn about the ancient Greeks as well as the Second Law of Thermodynamics. Is this model outdated?

  107. sarang Says:

    Some points of disagreement:

    1. This post is really about a hypothetical standardized test that doesn’t generate too many perfect scores (like the SATs) or too many zeros (like the Putnam) to be useful. This hypothetical test might be a unicorn. Maybe any test of reasonable length that is useful at truly sorting the top 3-5% of test-takers is like the Putnam for the average test-taker. (This would argue for Ivy-specific entrance exams but it is not clear how the socio-economics of these would play out…)

    2. I’m especially skeptical about the workability of the writing component, which (in its existing form) is dire. The essays are graded at the rate of more than one a minute by people who do not necessarily know what they’re doing. (Whether writing ability is well measured through timed tests is a separate question.)

    3. Something I found annoying about the Pinker article is that he uses evidence on general populations to make arguments about highly atypical subgroups (i.e., people applying to Ivy League universities). Certainly someone who is illiterate will flunk the SAT verbal and writing sections but this is not to the point. (Similarly I can predict US presidential outcomes state-by-state with about 80% accuracy without a minute’s thought.)

    4. Throwing out information is not necessarily bad, to the extent that it reduces the incentive to game the system (or to the extent that it makes the admissions process less stressful — it might also be that the higher the stress level associated with a test, the more performance on that test reflects qualities such as nerve that one should not be selecting for). I would conjecture that the distributions of “true mathematical ability” of people with a 750 and those with an 800 on the math SAT overlap heavily; the small amount of information gleaned from telling 750s apart from 800s must be weighed against these costs.

    5. To the extent that the smartest students are distributed over 10-20 universities rather than being concentrated at one or two, that’s not such a bad thing for society (or even, really, for the students themselves!). Again, the lower the prizes, the less the incentive to hustle/cheat. (Steve Waldman said the other day that you can think of inequality-promoting policies as “a tax on refraining from mercenary behavior.” A similar point applies here.)

    None of this is to deny that the modern-day professionalized admissions office is a depressing thing, or that professors should have much more input than they currently do. (Of course most professors would probably rather not be bothered, but perhaps they should not have a choice in the matter.)

  108. ScentOfViolets Says:

    Harvard claims to admit “special” applicants, not well-rounded ones. It is a lot more plausible about Harvard than about Yale.

    Another data point: back in the day (mid-70’s) I was accepted at MIT with a partial scholarship, despite not having a perfect SAT score. I suspect the difference between me and Scott is that I grew up on a farm with no electricity and no running water and had several hours of chores to perform every day before I could even think about homework of any sort. Just guessing here, but it’s not implausible to assume Scott grew up at least middle class, possibly upper middle class. The difference in outcomes lends credence to this sort of speculation.

  109. ordinary University old student Says:

    Scott: Since this is about education. I am from India but not from the IIT but from an ordinary school. I got into PhD in a big school in US with funding and the area is theoretical comp science. All the Indian people I see in the school I have been admitted seem to be from IITs and I am standing alone. You think an ordinary fellow can get a decent PhD from a top 15 cs program? I am also fairly old with lot of industry experience.

  110. Scott Says:

    ordinary University old student #109: Well, I guess it depends what you mean by “an ordinary fellow.” Since you say you’ve already been admitted to a major PhD program in theoretical computer science, you’re probably not that ordinary, at least by ordinary standards! 🙂

  111. Raoul Ohio Says:

    Michael Dixon #93:

    Universities are huge operations dealing with many competing tasks, somewhat rationally, but also hamstrung by political reality, doofus administrators, constant need to scrounge for money, accreditation, etc.

    You have a vision of what you want a university to be. Maybe it is a good idea, maybe not. In any event it happens to be very far from reality. Most of the people you talked to probably tried to point out how things actually work. You can try to find a path through the maze that works for you, or you can walk away. In most places you can get some leeway if you are smart enough to do well in whatever class you take.

  112. ScentOfViolets Says:

    Does anyone have any data about how much these institutions take into consideration a student’s background? I’m getting the sense from several people that they had, er, a privileged upbringing. I think the one thing we can all agree on is the last thing we need is more people being admitted on the grounds privilege. My daughter comes from a background of privilege, BTW: She was fortunate enough to have parents concerned about her education and who gave her immense quantities of attention when it came to anything academic. We also had the money to indulge her when it came to any subject that caught her fancy. Perhaps predictably, her grades suffered when she was no longer at home. Certainly she looked much better on paper than she was in actuality.

    I suspect this taken at least partially into account in the admissions process. Or at the least, hope it is.

  113. Scott Says:

    Sam Hopkins #106:

      Do you think students should be required to take a wide range of classes? Part of the liberal arts model as I understand it is that you’re supposed to learn about the ancient Greeks as well as the Second Law of Thermodynamics. Is this model outdated?

    I’m a huge fan of a broad liberal-arts education. Probably the most memorable courses I took my entire time in college were creative writing seminars. And I’m constantly encouraging my advisees at MIT to take some real liberal-arts classes, rather than fulfilling their liberal-arts requirements with econometrics or game theory or video-game design (which, yes, you can do at MIT 🙂 ).

    However, I think it’s crucial that students have a lot of flexibility in how they fulfill their liberal-arts requirements, so that they’re not forced into courses they hate, something that achieves the exact opposite of the desired broadening effect. (After a traumatic experience in 11th-grade English, it was years before I felt ready to read Shakespeare again—at which time I realized to my delight that, hey, Shakespeare’s actually pretty good!) Even though I do nudge my advisees toward broadening their horizons, another thing I’m constantly doing for them is signing the paperwork for them to substitute one course for another one, when they’ve clearly decided it’s what they want (I almost never refuse substitution requests).

    And, as I said, I think that admissions offices should be looking for the extremes, much more than for well-roundedness. By all means, try to expose the future Fields Medalist to some history and philosophy, and the future Pulitzer prizewinner to some math and science, once they’re at your university. But for godsakes, don’t reject those people!

  114. Scott Says:

    sarang #107: You raise some interesting points. But if it were really the case that every standardized test had either the problem of too many zeroes, or the problem of too many perfect scores, then I have a simple solution to propose: why not just use one test of each kind? 🙂 So for example, why not use the SAT for the vast majority, and the AIME or USAMO or some other math competition to differentiate the students at the very top?

  115. Parent of young math whiz. Says:

    (Continuing #95:) Jon #99 got me thinking. Was the real problem lack of extracurriculars (polo sailing soup kitchen etc) or was it lack of academic credentials in English, history, science, language.

    I’m prepared to say screw this EC nonsense and we’ll take our chances. But as a homeschool parent I’m concrened I can’t do justice to higher levels of English, history, science, language and might have to resort to B&M school (or otherwise outsource) for high school. I’m all for specialization (the opposite of American Unis) but I don’t want to push our luck too far.

    A financial point to make: Unis like Harvard are extremely affordable for those with modest incomes, and that is a major incentive to try to get into one of the few genuine “meet full (financial) needs” schools.

  116. eitan bachmat Says:

    + Scott 52
    Scott, your Hebrew is a lame excuse, I believe I personally saw you ordering a Shakshuka, so you passed the Israeli IQ test. Now if you can make a Shakshuka, you can also enter the prestigious programs, like medical school and law school, as is the case with Doctor Shakshuka or the inventor of the Shakshuka method (at least as useful as the sieve method or orbit method) Ram Caspi, one of the most famous Israeli lawyers whose name literally means “My money is more worthy”. Anyway, whats done is done, I will enjoy your blog regardless.

    + David 65
    I agree, the failure rate in Freshmen year in math was and still is roughly 50%, we even had recent years with 80% failure in calculus across several universities. But at 3K $ a year, it doesnt matter
    much, at least you learn what you are not good at. Israelis spend 2-5 years in the army, so 1 or half a fruitless year is rather meaningless (as an aside, I highly recommend the movie “zero motivation”).
    I personally started math/physics, got good grades in math, was really bad in physics which I dropped. I also dropped philosophy because I didnt like the studies and wasnt particularly good at it.
    Heck, I did a PhD and postdoc in pure math and still dropped out of academics in this field, I still consider this grand failure as the best professional experience of my life.

    If a tryout year in college isnt a 60K$ Disney production then people could fail in the first year and survive the experience, and universities would have the guts to tell people, hey, this is not what you were destined for, rather than failing 2 out of a class of 150.

    I am not saying this would work for the US, just pointing out some experiences.

  117. rrtucci Says:

    ordinary University old student:
    “You think an ordinary fellow can get a decent PhD from a top 15 cs program? I am also fairly old with lot of industry experience.”
    The answer is that you are going to waste your time. PhD’s are for people who want to be university professors. Is that what you want? If not, I would advise you not do a PhD at all. If all you want is to learn more CS, in this day and age, you can learn almost anything you want on your own through books, online courses, asking question in forums, etc. Hasn’t living in India taught you to be self-sufficient? Man, haven’t you heard of the Khan academy and MOOCs. Read my previous comments on MOOCs. As an Indian, WTF are you doing in America anyway. Your country is thousands of years older and wiser than piddling America. Go back to India and start an Indian computer science company that will rock the world!

  118. Gabriel Nivasch Says:

    The “top” schools are anyway ridiculously expensive. You can get an education that is just as good for a fraction of the price at a state university. Can’t you?

  119. Michael Dixon Says:

    @Raoul Ohio #111

    Oh don’t get me wrong. I quickly recognized that the reality didn’t match my initial expectations going into undergrad. I was fine with this since I hadn’t actually touched a university before nor understood any practical limitations they faced. This is why I talked to administrators in the first place. While it would be nice, I wasn’t really trying to change them or get my way. I wanted to hear, from them, what their intentions were and what they were doing to accomplish that.

    “Most of the people you talked to probably tried to point out how things actually work.”

    This, along with the motivations for such, is what I tried to get out of them. The trouble I faced was that they were unable to articulate the processes or explain the “why” of things. Why is their education designed better than a cookie cutter one? Why choose to do thing X this way over another thing Y?

    After 4 years of asking around I finally got someone to point me towards the university’s ABET accreditation. This says only a little on what the university is trying to accomplish to any specificity. I fail to see how it makes the university interesting, but someone there thinks it does. They were, of course, unable to explain why.

  120. Michael Dixon Says:

    Scott:

    The next MIT admissions cycle is approaching. Why not sit in and see how things work and what they are doing?

  121. fred Says:

    Seems to me that it all mainly depends on what the goals of universities are…
    Raise the average education level in the country?
    Nurture the very top of the cream of the intellectually gifted?
    Provide professors with talent pools in order to spin off their gig in the private sector and summon a few billions dollars?

  122. Parent of young math whiz. Says:

    Gabriel Nivasch #118, you are quite mistaken about the pricing of American universities. For the poorest 90% of households, the top private unis charge only a very small percentage of the sticker price, whereas state unis charge all of a high price to non-residents of the state (and most states don’t have a top notch state uni, so you need to cross state lines). For a typical income household (outside California) UC Berkeley could be at least 5 times more expensive than Harvard. It’s not just a question of education quality. There are huge financial stakes in trying to get into one of the few top notch unis like Harvard that are quite affordable for typical income households. If you don’t get in, then the only other affordable option is probably the in-state public uni which in most states is quite mediocre.

  123. Scott Says:

    Michael #120: I did read MIT admission folders in previous years, so I know a bit about the process.

  124. Scott Says:

    Gabriel #118: I think one of Pinker’s main points was that, for most undergraduates, the distinction between a “first-tier” and a “second-tier” university is indeed educationally irrelevant—they can get an equally-good education at either, so if it weren’t for the issue of reputation, they really should just shop around for the best value. The distinction is most relevant for a small subset of the students: the academically most advanced students who can benefit most from the top faculty. Yet those are often precisely the students who are kept away from the first-tier universities by “holistic” admissions practices.

  125. Ben Standeven Says:

    @Darrell Burgan:
    Sorry, I must have been confusing you with matt2977. The only problem with your suggestion that I see is that, at most colleges, admitting a undergrad student into one field means admitting them into every field. So they would want to see good test scores in every area, not just the ones the student intends to study. Having more areas that could be tested would make this problem worse.

  126. Douglas Levene Says:

    “I don’t really understand why none of the elite colleges took the opportunity to grow as the number of applicants grew enormously. Why didn’t Harvard open a California campus and a Texas campus? Why did every last one of them decide to become way more selective over the past 40 years instead of staying equally selective and growing?”

    Don’t blame the schools. Blame Messrs. Gates, Jobs, Dell, Soros, Buffet, et al. The last great wave of wealth creation saw the beneficiaries use their wealth to establish many great universities: Stanford, Chicago, Carnegie Mellon, etc. The current great wave of wealth creation has lead to the establishment of exactly zero great new universities. Why? I don’t know. Ask, Messrs. Gates et al.

  127. ordinary University old student Says:

    @177 rtucci and @110 scott I do want to be a prof and starting a company where there are talented and most importantly trained people is easier. India could have a lot of talent However the training here sucks. US is a country of superlatives. That is why I posted the comment on Scott’s blog on whether a student (of ordinary academic and cultural pedigree from an ordinary school and going for first PhD in even extended family) who has taken some tough math class work at grad level (yellow books) and undertook some individual study and very hard projects (but could not make much progress due to the problems long standing issues) from some Princeton and UChicago math Phd graduates who are now profs (and who also recommended for admission at the top 15 school) can compete with the young best and brightest of his country or other countries who have jumped through great exams and intellectual hurdles.

  128. Tyler Says:

    @Scott:

    You seem to be accepting of, yet displeased with, the “free market” argument. And honestly, I don’t blame you, and here’s why:
    While schools have a right to their own admissions process (and you, of course, have a right to gripe when they suck), what’s not okay is false expression of intent, as some have touched on above. It’s one thing to include non-academic criteria, but to do so and claim otherwise is lying. This is especially relevant to your suggested faculty-rights movement. If a school employs professors under the assumption that the professors’ classrooms will be filled with some type of student, then the effort should be made by admissions to supply that type of student.

    So if hypothetically MIT says it is looking for only math/CS kids and will focus on providing to them a specialized education in math/CS, and a music professor comes along and gets a job at MIT, and then complains when all the students in his studio are programming Space Invaders on their calculators, I personally would have no sympathy for that professor. But I see your call-to-arms justified when schools are not so clear/honest about their true intentions. If Harvard tells their faculty “We’re looking for the best and the brightest”, and then a professor asks the admissions officers about a particular student who is struggling, and the response is “Oh ya, in addition to the best and the brightest, we forgot to mention the kids whose parents are rich alumni”, by all means that professor should feel entitled to an admissions process with a higher weighting of standardized tests (or some other system more merit-based than this guise of “well-rounded” bs).

  129. jonas Says:

    Hello, Scott!

    I heared one problem in the U. S. with standardized tests is that they use them to assess the quality of high schools and high school teachers, without giving any incentive for the students to perform well on the test. (This model is since copied in Europe, but that’s irrelevant now.) Is this true? If it is true, how can you tell anything sure about what those standardized tests measure, and how would you bootstrap the standardized tests used for university admission to work well?

  130. Scott Says:

    jonas #129: I don’t know if that’s true, though it might be. I’ve also heard of the opposite problem, exemplified by the Simpsons episode where Mrs. Krabappel announces to her students before a standardized test: “the worse you do on this, the more remedial funding the school gets, so don’t knock yourselves out…”

    In any case, these aren’t problems with the tests themselves, but with the use of those tests by governments to reward or punish entire schools. They could be fixed by not using the tests in that way.

  131. Michael P Says:

    I wonder how would admission of, say, football players would go if the same policies apply to prospective athlete students: “We know you can play football on NFL level, and our football coaches want you on the team, but you don’t have any community service on your application, so we’ll accept a physically disabled math nerd instead for the running back position.”

  132. ordinary University old student Says:

    I do not understand what is the big deal about this college admissions process. America is doing good. There is one fundamental issue every facing multiethnic nation – addressing more equitable representation of colleges by demography. There is a system (high school system) which produces output whose quality depends on the nature of the input (race, economic situation, social situation). The output gets consumed (college admission) based on some criteria. Since the output qualitydepends on the nature of input, one needs to ch

  133. ordinary University old student Says:

    ….one needs to change to the system to make the output more equitable. Other than that US educational system is fine. The admission process is varied depending on the school in many countries where one or two tests decide your fate. I think this variety is good for the nation as a whole. I doubt if academics was the only

  134. ordinary University old student Says:

    ….criteria, you would be seeing the potential of humans to capitalize on fundamental research done at various institutions possible. Even the best IITians come to the US and then succeed because there is something that is ingrained in US society that follows through to colleges and flows back to society.

  135. Jim Says:

    @Scott 130:

    I have never heard of a Krabappel situation occurring in an American school system. If anything, the opposite has been true for some time in which teachers have been forced into impossible situations where they’ve felt they had to cheat for their students to get the necessary money, e.g. in the Atlanta cheating scandal: http://www.newyorker.com/magazine/2014/07/21/wrong-answer

    Also, if you were curious about whether the tests (or rather the models associated with the tests) were bad or not, wonder no further: http://mathbabe.org/2012/03/06/the-value-added-teacher-model-sucks/

  136. Richard Says:

    Scott wrote
    ” (Having said that, going abroad is certainly another option, and I’ve heard that this Cambridge University is a real up-and-comer on the academic scene. 😉 )”

    Funny you should say that – here in the UK we have some similar problems with Oxford and Cambridge (a disproportionate number of those accepted seem to come from elite private schools) and the remedy often proposed is to apply to an American University.
    There was a well publicised case a few years ago of a very well qualified (on paper) student who did not get in to Oxford and instead was accepted by….. Harvard!

  137. Parent of young math whiz Says:

    Question: If a kid is a USAMO participant but otherwise doesn’t have much going for them (apart from being educated), which are the top institutions that will admit this type of kid, and which ones won’t?

  138. John Sidles Says:

    Ed Witten and I have little in common, but we do have in common an affinity for algebraic dynamics, and spouses who have served on urban school boards

    Over lunch (many years ago), Witten opined that, in his experience, the difficulties of the latter had proved to be considerably greater than the difficulties of the former.

    The facile-yet-inchoate educational fixes offered by Prof.  Pinker (and here on Shtetl Optimized too) are a testimony to this modern-day complexity hierarchy.

    Rather than say more, I’ll attempt to distill my opinions into an integrated set of essays that plausibly will elicit banishment a few more times from Shtetl Optimized (as has been foretold).

    Now that is the way to write — peppery and to the point. Mush-and-milk journalism gives me the fan-tods.

      — Mark Twain
         &nbspJournalism in Tennessee (1871)

    ——-

    Self-Teaching Exercise for STEM Students

    (1) Go to the lobby of any STEM department on your campus.

    (2) Find a plaque documenting year-by-year awards to “The students most promising of an outstanding career in academic research” (or any other anticipated outcome)

    (3) Research the subsequent publication history of those students.

    (4) Assess the predictive capacity of University Admissions-and-Awards committees.

    (5) Discuss Ed Witten’s career trajectory in light of these findings.

    Note  This exercise can be illuminating for STEM faculty too.

  139. John Sidles Says:

    Noah asks “I don’t really understand why none of the elite colleges took the opportunity to grow as the number of applicants grew enormously.”

    It is a pleasure to suggest a student assignment that answers to your question, Noah.

    Exercise  Verify that there is a sharp “knee” in the curve of reputation-versus-endowment that is located (approximately) at $250,000 per student.

    Example  Cornell University, for example, is located precisely at this knee; hence struggles to be recognized as a top-rank school).

    Assignment  Critique the following assessment:

    • It’s economically difficult for low-ranking universities to approach the endowment-knee; hence elite universities are not born.

    • Universities above-the-knee are governed as corporations — literally, the Harvard Corporation, the Princeton Corporation, the Stanford Corporation, the MIT corporation, etc.s — these corporate boards elect not to grow for the same reason that MicroSoft Corporation elects to sit quiescently upon nearly one hundred billion dollars in cash.

    Thank your for your fine question, Noah!

  140. Michael P Says:

    ordinary University old student #132-134:

    1. America is not “doing good”. Compare measurable math and science ratings of American students with students of other countries, compare crime and incarceration rates, etc. Here is a glimpse into what Times Magazine editors think of average Americans’ abilities to discuss serious issues, as compared to the rest of the World:

    https://www.google.com/search?q=time+magazine+covers+different+in+us&biw=1295&bih=549&tbm=isch&tbo=u&source=univ&sa=X&ei=QF4YVJ7lFMijyATN1oDoBQ&sqi=2&ved=0CB0QsAQ

    2. Educational system if very far from fine, at least on high school level, as compared to other developped countries. IMHO a big part of the problem is precisely the desire to have the impression of equality by setting academics bar very low to erase the distinction between mediocre and exceptional. Here’s an example of such practices outside academia:

    http://usnews.nbcnews.com/_news/2012/02/09/10362607-white-firefighters-awarded-25-million-in-discrimination-case

    3. Criteria for choosing university students based on things unrelated to their academic abilities is like choosing football players based on criteria unrelated to their physical abilities in my post above. Dismissing the candidates’ ability to do what they are going to be expected to do as the measure of their expected success is silly at best.

  141. aravind Says:

    Dear “ordinary University old student”, several Indian PhDs (and others) do quite well in the industrial/academic system in the US and elsewhere, whether they are from the IITs or not. (I speak as a US professor of (theoretical) computer science, who got my undergrad degree at an IIT.) I can understand your feeling of isolation, but the ground truth is that motivation counts at least as much as one’s undergrad background — and you likely have significant motivation, depending on the choices you have made. I wish you the best; please email me if you want to discuss further.

  142. Mark Srednicki Says:

    I’m really shocked by the story of Jon’s kid (and also Scott’s personal experience). I had thought that schools like Berkeley and (especially!) Caltech were still open to high achievers who had not necessarily bothered to check-off all the “well-roundedness” boxes.

    Here at UCSB we do have an option for these kids: the College of Creative Studies (CCS), which runs high-level undergrad programs in Art, Biology, Chemistry, Computer Science, Literature, Math, Music, and Physics. I’m familiar with how the Physics program works, and I know that admissions are handled by physics faculty who are thrilled to find prodigies! Furthermore, in CCS, students construct their own programs of study (in close consultation with faculty advisors) and have no specific general-education requirements.

    I guess I had always just assumed that every high-level college had an equivalent mechanism. Very sad to learn that it’s not so.

  143. Rahul Says:

    I think we are creating a false dichotomy in the analysis. SAT versus current admissions committees.

    I think the key problem in the current process is the Admissions Office. I can live with a system that has a low emphasis on standardized tests but instead gives the relevant Professors more discretion.

    e.g. Let the Physics Prof select the Physics majors. They can always select a student with a crappy SAT score just because he shows some other promise (maybe he helped rebuild his college physics lab!) A stellar community volunteering experience might count for Psych admissions but hardly relevant to math.

    Conversely someone may have fantastic SAT scores but show just so much abject lack of horse sense that an Engineering Professor might think of him as not a good candidate.

    My point is that the current admissions committee is selecting on attributes that don’t matter much. They don’t have much of a clue as to what works and what doesn’t.

    OTOH, the big practical problem is that American admissions aren’t tied to a major at the time they enter. That makes my prescription impossible to implement. Maybe this itself is a fatal error. To think that you can admit a “good” student and reject a “bad” one with no reference at all to what he intends to major in. There’s no such thing as a generically good or bad admit; it all depends on for what major.

  144. David Sanders Says:

    In reply to Matt2977 Comment #21:

    Agreed. I’ve previously enjoyed some of Scott’s blogs, particularly for their ability to dance on the line between rant and reason. But this one is too much of a rant. I suppose it’s always disappointing to see someone engage in arm-chair sociology, especially when they’d tend to benefit from their prescriptions. I feel I was bitten by my inability to measure up in standard units. I guess there’s something about tests and grades that are particularly toxic to a depressed teenager. There’s also no more of an unpleasant thought than to see yourself as having been labeled “inadequate” by society. What better way to facilitate this than to slap a score on each person?

  145. Mattie Says:

    “Well Roundedness” as define by “Jon’s Kid” means meeting basic academic criteria of passing a broad set of prerequisites. It does not mean fancy admissions essays, sports you hate, and spending 10 hours a week at “charities” you don’t care about. It is absolutely baseless to confound those two.

  146. Peter Says:

    Richard@136 – there was indeed a case of someone who got rejected from Cambridge who got accepted at Harvard – she got rejected to study medicine, she got accepted to study biochemistry. I don’t know how things are in the USA but in the UK at the time the entrance requirements for medicine were a lot tougher than for biochemistry.

    Also if I recall right she was an independent school pupil and felt she was a victim of reverse discrimination; this runs counter to the usual complaint. One issue with Oxford and Cambridge is that state school pupils are less likely to apply than equivalently-qualified independent school pupils, the great motto of the admissions tutor is “we can’t let you in if you don’t apply”.

    I went to Cambridge from a state school; for a brief while I thought I wouldn’t, for inverted snobbery reasons, but when I got a prospectus in my hand all of that silliness went away and I never looked back. I’m not sure that everyone can overcome their classism like I did, one would have hoped that people clever enough to get into Cambridge would be above such things but alas not always.

  147. Michael P Says:

    Mattie #145: One of the problems is that “basic academic criteria” one is supposed to pass a very basic indeed, and essays and sports and charities become the differentiators.

    To make things worse the criteria for scoring essays and extracurricular activities are often odd as well. I was asked to score applicants’ essays a couple of times, and was supplied with 7 criteria and certain rules how to score on those criteria. One of the criteria was the *number* of extracurricular activities; another one was a measure of involvement, mostly time spent correlated; three criteria were related to essay itself rather than the information it contained, such as grammar and clarity; another one was the applicant’s character that I have no clue how to measure from reading an essay, and finally, lo and behold, was a single item about the relevance of the applicant’s activities to his chosen career path.

    Under these rules, with only 1/7th of the equally weighing criteria highly relevant to the student’s success, how one could expect anything good coming out of primarily essay-based admission process?

  148. Itai Says:

    Wow,
    Seems like a hot debate.
    I think a balanced solution is to do things like in Technion, Israel.
    Most important Sports and etc do not count for admissions.
    However,you must do 2 courses in whatever sport you like .
    and up to 10 more courses.
    Also you can be in a team and get 100 for sure +benifits like doorms and more.
    Also even chess team is considered sport.
    Also there are many different “sports” like snooker , bowling, yoga, air gun, wall climbing and many more.

  149. Arko Bose Says:

    In India, most reputed universities and institutes conduct nationwide tests. I do not see why US universities, which have much larger funds and much smaller number of applicants, could not move to such a simple and indisciminatory mode of admission.

  150. Oliver Says:

    I admit that my views on this matter might be colored by my strange (though as I’ve learned, not at all unique) experience, of getting rejected from almost every “top” college in the United States, and then, ten years later, getting recruited for faculty jobs by the very same institutions that had rejected me as a teenager.

    As some commenters have noted, this behavior on the part of the ‘top’ universities is not necessarily contradictory, but rather indicates that their admissions criteria are not structured to favor future academics. This is not particularly surprising on the part of Harvard, Yale, or Princeton, which have described their mission as training ‘future leaders’ for some time now. Producing graduate students may be seen as a secondary function by private universities that can receive greater ROI from future tech entrepreneurs or financiers. It does not appear that Harvard’s admissions criteria have disadvantaged them in this market, although they now face more competition from Stanford. MIT/Caltech/Harvey Mudd have historically captured much of the STEM prodigy market, and can afford to be very selective in this stratum. Around the time you were applying to college, Case Western Reserve University was making a serous play for the disadvantaged high-SAT cohort by offering scholarships up to full tuition graded entirely according to SAT scores; University of Chicago also offers merit scholarships based in large part on test scores. Case has produced some very successful graduates in the biomedical and technology industries using this strategy, and has also attracted private money for its endowment. Cornell is a top university with a greater emphasis on engineering and applied sciences than HYP, and fills a niche between HYP and MIT/Caltech. CMU is a longstanding CS haven – these two schools would be a good fit for a precocious STEM applicant.

    I don’t mean to be an apologist for the system, which has many serious and possibly fatal flaws, but I wanted to note the heterogeneity in school foci and selection criteria. From what I have observed, I think Case Western Reserve University and UChi (along with institutions like Reed College) might have come close to passing the ‘professor test’ at various points in the recent past. I think part of the onus falls on students and society to choose to attend and elevate that status of those universities having admissions policies reflecting their values. I also do not think that the undergraduate selection criteria are intended to resemble the faculty selection criteria – the pipeline is meant to be leaky at most universities.

  151. Dave R Says:

    Scott #105: Point of information, Cornell actually is an ivy 🙂

  152. Scott Says:

    Dave R #151: Yes, I’m well aware. 🙂 Fixed; thanks.

  153. Liz Says:

    Some of the commenters said that the SAT produces too many perfect scores. There are actually only about a hundred 2400’s a year and a thousand scores of 2370 and above, so people scoring 2370+ are about the top 0.07%. (At least that was the case in 2009, when I scored a 2370 and got rejected or waitlisted at a few top schools and got into MIT.) Sure, the difference between a 2350 and a 2400 is a few inconsequential mistakes, but only about 2000 students a year are mastering the SAT’s subject matter, so this isn’t quantitatively a huge issue.

    I think that even if universities want to optimize solely for academics, they should try to admit people who are helpful, friendly, and have good social skills, because these students are likely to help their friends learn, be productive, and survive the stress of being at an academically intense school. MIT, where I interview undergrad applicants, says it prioritizes these things, but I don’t know to what degree it can actually identify nice people.

    More on interviews: Although it’s hard to tell if I’m good at reading students’ personalities through an interview or whether the admissions committee listens to me, I’ve definitely identified several students who, upon being pressed, couldn’t say much about the things they claimed to be really interested in, and I flagged a few who seemed like genuinely nice and helpful people. It seems like this process works — the people I met as an MIT undergrad were, by and large, incredibly nice and friendly people. This is much less true of the grad students i met at MIT or at Harvard, where I’m currently a grad student, but that may have more to do with the structure of grad school than with the people.

  154. Michael P Says:

    Liz #153, IMO universities’ primary goal should not be being a social club, and academics should matter more than social skills. In particular, I am astonished that anybody scoring in top 0.07% could get rejected by any self-respected university.

  155. Leo Reyzin Says:

    Here’s an argument against a test-scores-only approach (heard from people who have thought deeply about admissions): if you admit people who all measure themselves solely on a single scale (academic test scores), then they will continue measuring themselves on the same scale, except that now half of them will be below the median and thus miserable and, eventually, unmotivated.

    If, on the other hand, you admit sufficiently qualified students who excel in different ways, they find their groups and continue excelling in their different ways and stay happy and motivated — even the half that’s below the median academically. In other words, I think Pinker underestimates the social dimension of the college experience: it is important to consider how the conditioning of the pre-admissions high school life will manifest itself in the post-admissions college life.

    I emphasize that it’s not an argument for the keeping the status quo.

  156. Michael Dixon Says:

    #154 Michael P

    You seriously need to reread Liz’s comment (almost feels like you didn’t). She talked about the hypothetical scenario that is equivalent to your opinion. She explained why she felt the social aspect was still important.

    The SAT isn’t a fantastic measure or indicator about academic potential. It doesn’t control for a lot of environmental variables nor is it encompassing. I don’t know how it could astonish anyone that someone with a perfect score could get rejected. Human beings are far more complicated than that. The metric is not sufficient to assess them.

    Even if you think that it is a solid measure, its clear from the existences of these rejections that the universities do not share this view. So again, it shouldn’t be astonishing nor surprising.

  157. Vaughan Pratt Says:

    There was a well publicised case a few years ago of a very well qualified (on paper) student who did not get in to Oxford and instead was accepted by….. Harvard!

    One pictures a score of reporters sitting around for a few years waiting for something interesting to write about, and this being the best they could come up with in all that time.

    This was the only example they could find of such a thing during those “few years”? Remarkable.

    As Scott points out there are a number of tenured faculty at institutions that rejected them as students. I’d be interested to know how many of them have been tenured at two institutions both of which rejected them as students. Examples, anyone?

  158. Oliver Says:

    There are more SAT takers each year scoring in the top 0.07% than there are slots in MIT’s entering class, and because students apply to multiple schools, MIT probably attracts a disproportionate number of these applicants for whom MIT is not the first choice. This would be a relatively large cohort of lower-probability acceptances, and MIT probably does a lot of complicated bet-hedging in this stratum to maintain its yield statistic.

    What is interesting to me is that it appears these high-scoring candidates continue to express a strong preference for universities known to place significant weight on factors other than test scores, even between similarly-situated institutions. Does this suggest that they also value the ability to network/interact with students having these characteristics? Why is reputation so robust? Many of the the most successful universities (in finances and reputation) seem to have adopted the strategy of making certain concessions to attract students reflecting the values of society in general (charisma, social facility, resources, status, attractiveness), which in turn helps them to attract a larger cohort of high performers. If you can maintain this strategy, the academic concessions you have to make probably decrease with time.

  159. John Sidles Says:

    Oliver asks  “Why is reputation so robust?”

    Observation As well to ask “Why are million-dollar-per-student endowments so robust”?

    After all, the correlation between elite academic reputation and elite levels of per-student endowment is sufficiently strong, that the two questions are operationally identical.

    Answer  It is rational for the governing boards of elite university-corporations to regulate student enrollment so as to ensure a secular increase in per-student endowment … and hence a secular increase in institutional reputation.

    Postulate  The artificial scarcity of perfect-SAT students is a fig-leaf that cloaks this academic-corporate strategy in faux-meritocracy.

    It would be terrific if pundits like Stephen Pinker directed their analytic talents toward these economic observations and corporate behaviors!

  160. John Sidles Says:

    Parallel readings  21st century STEAM students are invited to consider whether the Pinker/Deresiewicz debate echoes the main themes of the celebrated 20th century Jaffe-Quinn/Thurston debate.

    References  Jaffe-Quinn’s “‘Theoretical mathematics’: Toward a cultural synthesis of mathematics and theoretical physics” (see arXiv:math/9307227 [math.HO]) and Thurston’s reply “On proof and progress in mathematics” (see arXiv:math/9404236 [math.HO])

    Pinker’s argument summarized in Jaffe-Quinn’s language “Speculative mathematics admissions policies are dangerous; serious caution is required; objective proofs are preferred.”

    Deresiewicz’ argument summarized in Thurston’s language (per Thurston’s “Crucial forms of progress in mathematics are non-objective.”

    Much more has been written, needless to say … however, the verdict of STEAM history has already been delivered:

    Verdict  Jaffe-Quinn (and Pinker) present the more brilliant arguments; yet Thurston (and Deresiewicz) present the more nearly right conclusions.

    Conclusion  Yes, Scott is right to evaluate Pinker’s essay as “brilliant”, however the history of mathematics shows us plainly — in multiple ways! — that doctrines of dominant brilliance constitute insufficient grounds for concluding that Pinker’s conclusions are good ones.

  161. Michael P Says:

    John Slides #160: IMO the proposed analogy between university admissions and mathematical progress is false. For once, mathematicians (hopefully) are not in the business of intentionally obfuscating their goals. IMO a better description of the current university admission process is much better described in the definition of [truthiness](http://en.wikipedia.org/wiki/Truthiness).

  162. Michael P Says:

    For example, the original post refers to two historical discriminatory agendas of “holistic” admission policies, which were certainly not explicitly proclaimed. I am not aware of any work in mathematics that was intentionally hiding the intent.

  163. Rahul Says:

    The other general point is that neither SAT nor the holistic admissions committee approaches etc. can ( nor should ) be designed for the exceptions.

    A 14 year old hyper-performer is an exception. I’m not saying he shouldn’t get a chance. He should. But that should be as an exception.

    The system shouldn’t be so rigid that exceptions cannot be accommodated but nor can we design a good system by basing it on the outlier cases.

  164. John Sidles Says:

    Michael P “Mathematicians (hopefully) are not in the business of intentionally obfuscating their goals.”

    Between “intentionally obfuscating” and “scrupulously illuminating” there exists a vast domain of discourse that is rapidly being colonized with creative works both passionate and wonderful (often both).

    Three student-friendly examples (from among hundreds) are:

    •  Bill Thurston’s “On Proof and Progress” (1993), and

    •  The Grothendieck-Serre correspondance, (2007) and

    •  Donald Knuth’s “Let’s Not Dumb-Down the History of Computer Science” (2014).

    Conclusion  As 20th century’s STEAM-fogs of trade-secrecy, guild-secrecy — including academic cryptolects — and state-secrecy inexorably dissipate, the social constructs that are destined for transformation surely include the construct that Pinker analyzes … academic gatekeeping!

    $latex \scriptstyle\rule[2.25ex]{0.01pt}{0.01pt}\,\boldsymbol{\overset{\scriptstyle\circ\wedge\circ}{\smile}\,\heartsuit\,{\displaystyle\text{\bfseries!!!}}\,\heartsuit\,\overset{\scriptstyle\circ\wedge\circ}{\smile}}\ \rule[-0.25ex]{0.01pt}{0.01pt}&bg=bbffcc&fg=0055ff&s=2$

    morediscourse@tradermail.info
    A fan of *MORE* discourse

  165. Live & Learn Says:

    Michael P,

    If you hadn’t already realized it, you will soon learn that John Sidles is loath to adjust his position as a result of a (non-accredited?) comment or reply.

    Go back and read his many posts — it’s a difficult task to locate any that say, in effect: Oh my. I hadn’t considered that point, or given it enough emphasis. That really does change my thinking on this matter.

    Now, of course, many others are also guilty of this . . . but at least our host does from occasionally explore that humble territory. 😉

  166. John Sidles Says:

    Live & Learn asserts “John Sidles is loath to adjust his position.”

    By design  The citations of my comments encompass sufficient ambiguities and even contradictions, as to ensure that there is scarcely any need for adjustments.

    With intent  We can all of us help to point STEAM students — aka TEAMS students — toward creative works that escape the coarse net of undergraduate course-syllabi.

    As authorization  Donald Ervin Knuth’s recent Kailath Lecture, “Let’s Not Dumb Down the History of Computer Science”, (2014) provides plenty!

    Example  As a friendly example of “smartening-up” our appreciation of STEAM history, let’s reflect upon the epigraph — an exceptionally well-chosen epigraph (as it seems to me) — of Andrea Barrett’s recent novel-of-science Archangel (2013)

    “We cannot part with our friends. We cannot let our angels go. We do not see that they only go out that archangels may come in. We are idolators of the old.”

       — Ralph Waldo Emerson (1841)

    Background  Andrea Barrett and Rebecca Goldstein (see appended references) treat very similar subjects from greatly differing perspectives; both authors are highly recommended to Shtetl Optimized readers (and Scott like me is a Rebecca Goldstein fan).

    Barrett’s epigraph, together with the illumination of it that Barrett’s novel provides, help us to appreciate the closing lines of Leila Schneps’ perceptive review (long version) of the AMS’ Grothendieck-Serre Correspondence (2007)

    The difference between them [Jean-Pierre Serre and Alexander Grothendieck] might be expressed by saying that Serre devoted his life to the pursuit of beauty, Grothendieck to the pursuit of truth.

    Barrett’s work helps us to appreciate that an even better summary of the Serre-Grothendieck correspondence might be “Serre pursued the beautiful truths of the angels; Grothendieck pursued the radical naturality of the archangels.”

    Resolved for purposes of debate: The angel-versus-archangel themes of the Serre-Grothendieck correspondence come alive for Shtetl Optimized readers in the Harrow-Kalai Quantum Computing Debate.

    Summary of the arguments  Aram Harrow manifestly on the side of the (orthodox) quantum-angels, who worship within “The Church of the Larger Hilbert Space” Gil Kalai manifestly is on the side of the (heterodox) quantum-archangels, who embrace the broader dynamical vision of Grothendieck and his numerous 21st century heirs (e.g. Jacob Lurie).

    Herman Melville’s quantum overview

    “How many [quantum students], think ye, have likewise fallen into Plato’s Dirac’s honey head, and sweetly perished there? […] When on one side you hoist in Locke’s head Feynman’s head, you go over that way; but now, on the other side, hoist in Kant’s head Grothendieck’s head and you come back again.”

    Continued best wishes for enjoyable readings, smartened-up appreciations, and overall higher STEAM free energy, are extended to all Shtetl Optimized readers!

    % Andrea Barrett and Rebecca Goldstein

    @book{Barrett:2013aa, Author = {Barrett, Andrea},
    Publisher = {W. W. Norton}, Annote = {contemporary
    Rebecca Goldstein}, Title = {Archangel: Fiction},
    Year = {2013}}

    @book{Goldstein:1993aa, Address = {New York
    {$[$}u.a.}, Author = {Goldstein, Rebecca}, Publisher
    = {Penguin}, Title = {The mind-body problem : a
    novel}, Annote = {spouse Stephen Pinker,
    contemporary Andrea Barrett} Year = {1993}}

    % Knuth “Let’s not dumb-down …”

    @unpublished{Knuth:2014aa, Author = {Donald Ervin
    Knuth}, Month = {May 7}, Note =
    {\url{http://kailathlecture.stanford.edu/
    2014KailathLecture.html}}, Title = {2014 Kailath
    Lecture: Let’s Not Dumb Down the History of Computer
    Science}, Year = {2014}}

    % Schneps on the Grothendieck-Serre correspondence

    @unpublished{Schneps:2007aa, Author = {Leila
    Schneps}, Note = {Unpublished long version of
    \cite{Schneps:2007ab}, on-line at
    \url{http://www.math.jussieu.fr/~leila/corr.pdf}},
    Year = {2007}}

    @article{Schneps:2007ab, Author = {Leila Schneps},
    Journal = {Mathematical Intelligencer}, Number =
    {4}, Pages = {1–8}, Title = {A review of the
    Grothendieck-Serre correspondence (short version)},
    Volume = {29}, Year = {2007}}

  167. eitan bachmat Says:

    OK, I know everyone moved on to other issues like the microsoft thing, but I think I might have figured this one out.

    Harvard/Princeton admissions should work as before and moreover admit everyone they “like”, future leaders, well rounded, sons and daughters of, future rich people etc.
    Upon addmision, these folks will get all the social and fringe benefits of Harvard students, in fact they will also get a “graduation certificate” that certifies that they are qualified to work as a consultant or to run their parents oil empire. You will not have to study at Harvard and you can do your networking on a yacht in San Tropez.

    Separately, there will be a none admission procedure which upon acceptance, Harvard will not admit to accepting you but you will actually study at Harvard with the best professors under the sun. You will also graduate but as a basic member rather than a platinum member. This will say that you are qualified for a future in academia, industry, innovation,…, but you will not be allowed to be a consultant nor an MBA, since you have not been admitted.

  168. Ian Says:

    Lovely conversation here. Scott, I’d like your view on the following idea, which I haven’t seen anyone mention yet: Given what we now know about the timeline for the development of the teenage-young adult brain, intelligence, faculties of judgement, and emotional IQ, how about we admit that admitting anyone to “higher ed” at the age of 17 is insane? What about national youth service until 25 or so, and then maybe a round of SATs for 25-27 somethings, who might actually be ready to learn something?

  169. Scott Says:

    Ian #168: I confess I find that a repugnant proposal. How many scientific breakthroughs are produced by people in their 20s, and would be lost forever if we forcefully conscripted all twentysomethings into “national youth service”? (Israel does, in fact, conscript most of its 18-year-olds into 2 years of army service for women, and 3 or more years for men. And that does create serious problems for academically-minded Israelis, putting them at a competitive disadvantage relative to their peers in other countries. But it least Israel has the excuse that, for obvious reasons, it needs a conscription army.)

    Personally, I would do the exact opposite of what you suggest: I would let people go to college at whatever age they’re intellectually ready for it, even if that age is 10 or 11. And a typical age for starting college might be 15 or 16.

    This would actually achieve three salubrious goals at once: it would allow young people to enter the workforce and start contributing to society years earlier than today, with huge benefits to the economy. It would let young people (especially women) balance work and family more easily: they could finish school, have kids, and then still be in their mid-twenties, with plenty of time to start a career. Finally, it would save millions of young minds from being deadened by enforced busywork and routine—a fate they face today, not because they were convicted of some terrible crime and sentenced by a judge, but just because they haven’t warmed a chair for a prescribed number of years.

    Needless to say, I don’t expect anything like this ever to become reality.

  170. jonas Says:

    @Scott: some part of this already exists. People can enter collage earlier than normal if they take exams earlier. Almost no people do that though, probably because there are no rewards for doing so. People can also enter collage the first time later than usual, though the few people who do this don’t do it because they hadn’t been intellectually ready, but mostly because they couldn’t afford to go to collage earlier.

  171. aviti Says:

    Ian#168, let people go to college even if they are 5 or 6 as long as it is what they can do and they do it better. I wish there were a day when a kid that is precocious can be admitted immediately, unlike now when the few who try get a frowned upon.

  172. MattQPG Says:

    If you’re still reading this post – I would suggest taking a look at this op-ed about how someone’s experience with precisely the system you’re suggesting.

    http://yaledailynews.com/blog/2014/10/03/deresiewiczs-irony/

    I can say personally I felt the same way going through the ACT/SAT/AP/National Math Exam etc. wringer – completely burnt out on academics and everything really.

  173. Buckyball Says:

    Op-eds and blog commenters bellyaching about college admissions standards are thick with irony, in that their perceptions and reactions have absolutely no basis in science or empirical analysis, and rather are the result of gut feelings and what just sounds good.

    I have no idea how well legacy admissions does or doesn’t work at universities that do it, but 20 years ago I worked in admissions at a major public university and can shed some light on how real admissions processes work at universities that don’t do the legacy stuff.

    TL;DR: What you think would be ideal criteria is just hot air if it isn’t backed up with hard evidence, because the evidence sometimes turns out to show the opposite result than you would expect.

    This university was state-funded, and the state’s board of regents mandated that state-funded schools must admit a certain number of students (which at this school was less than the number that apply, hence competitive admissions criteria), and that they must grant degrees to the people of the state generally (so the student body must be diverse on graduation day, not just the first day of freshman year). So a faculty committee sets and annually refines the admission criteria based on empirical evidence and statistical analysis, and they try to predict “success” as defined by when the applicant, if admitted, would complete the requirements for a bachelor’s degree in a reasonable amount of time. In other words, when deciding who to admit to college, past performance only matters to the extent that it impacts future performance.

    The upshot of this is that only factors which have been proven at that institution to have a meaningful, positive impact on a student’s success are taken into consideration, and the factors are weighted according to how much impact they actually have. For students coming in from high school, the #1 predictor of their degree-completing “success” was in fact their class rank—and if it isn’t top-tier, it’s weighted by a rating for what school they went to (if in-state), because the lower the class rank, the more the quality of the school makes a difference. The school quality is determined by how well students admitted from that school in the last decade tended to do at the university. The #2 predictor, IIRC, was performance in a college-prep curriculum, i.e. the more pre-reqs the kid got a C or better in would help that much more, or hurt if they didn’t have enough.

    The actual impact of standardized test scores was almost zero. Low scores didn’t hurt, and very high scores could help, but they were only the deciding factor in a tiny sliver of borderline cases where the class rank was lower than average and test scores were higher than average. It was so seldom, I personally would not advise anyone to waste money and effort on those tests unless they’ve talked to an admissions counselor at the university, are borderline not gonna get in, and they’d do very well on the test. And even then I’d question whether it’s really necessary to seek admittance directly to the university as a freshman in the fall.

    See, at this school, admission criteria varies depending on when the applicant wants to start. Competition is highest and criteria the toughest if they want to start in the autumn after they graduate from high school. If they can wait till the following spring and just work & save money in the meantime, it’s much easier to get in. And, if they go to the local community college for 2 terms, then everything they did in high school no longer matters; they only need a 2.0 college GPA to transfer to the university, as the biggest success predictor for transfer students is their recent performance at an accredited college. So, applicants too far below the academic cutoff are encouraged to go to another college for a semester or three and transfer as a sophomore or junior. Applicants just slightly under the cutoff are encouraged to either do that, or to start in the spring as a freshman if they qualify. (At the community college, they’ll have smaller classes and sometimes the same professors at the community college as they would at the university, just with fewer extra services and non-academic perks. Oh and it’s cheaper too! But so many people think, erroneously, that they have to start and finish at the same school. Parents want to brag, you know.)

    Applicants who want to start as freshmen and who are just below the academic cutoff could also get in if they included a short essay that might put them over the line. There’s a subjective element (someone has to read it), but on the whole, the essays fall into certain categories of explanations for the kid’s sub-par academic performance. These excuses are quantified and scored based on rigorous statistical analysis. By far, the best thing a kid could write was that they’d been through some rough times and made bad decisions but were now highly motivated, had turned their life around, were the first in their family to go to college, have something to prove, etc.—i.e., expressions of high levels of motivation correlate very highly to future academic success. The most common thing I saw, but also the one worth the bare minimum statistically, was trauma due to divorce, physical health problems, or a death in the family. Major mental health issues scored slightly higher but still were in the “sh*t happens” category if they were just mentioned without the “I’ve got it under control now” / “I’m so motivated” followup.

    This was a large university with many services that the kids generally don’t have access to in high school, so if application or essay makes it sound like the student qualifies for support from (e.g.) university offices that specialize in helping minorities, kids with disabilities, etc., and they’re just a hair shy of the academic cutoff, then the application gets flagged for review and possible contact by the appropriate office. The office would figure out if there are other factors that need to be taken into consideration. Quite often they’d get pulled in for a learning disability test. You’d be shocked how many bright kids struggle all the way through high school because they had no idea they’re dyslexic or whatever and will actually do great in college with different study habits and (e.g.) extra time on tests. Then with an endorsement from whatever office, it puts the applicant over the top and they get to start in the autumn, and they end up doing just as well as the “regular” kids who barely got in on class rank alone.

    This system worked really well. Maybe it’s different now, but given the glacial pace at which change happens in large bureaucracies, I doubt it.

  174. Kaleberg Says:

    “I admit that my views on this matter might be colored by my strange (though as I’ve learned, not at all unique) experience, of getting rejected from almost every “top” college in the United States, and then, ten years later, getting recruited for faculty jobs by the very same institutions that had rejected me as a teenager.”

    Surely something happened in those ten years. I’m willing to bet you took a bunch of courses, found something you were good at and interested in, did some major work on it and a bunch of other things.

    —-

    Michael Dixon – 93:

    I think that girl who was enthusiastic about rocks was an actual answer to your question about educational philosophy at MIT. That was why I chose the school over Harvard, Princeton and Columbia, the enthusiasm. It permeated the place. (Harvard actually did cultivate enthusiasm, but in humanities nerds: the opera freaks, the language geeks, the history nuts, the literature jocks and so on. My high school had more STEM types, but lots of the humanities types as well.)

  175. Circe Says:

    A rather amusing side-story to Prof. Pinker’s call for greater emphasis on test scores in the US is that in India—where admissions to the most sought after science, engineering and medicine programs are based almost entirely on test scores—there has been a lot of discussion on how to move to the “American” system of “essays and letters”. All of this reminds me of an old Hindi saying, which translates roughly to “Drums from afar always sound more pleasant”.

  176. Links: Bogus authenticity, radicalizing the romanceless, universities, destiny, and more « The Story's Story Says:

    […] A totally indecent, untraditional, radical proposal: “universities should prioritize academics.” This may also ameliorate some of the current elite university admissions […]

  177. Yuval L. Says:

    Sorry to dig up an old thread, but I found this story interesting.

    I was in the same situation as you in high school and only went to a large state university. I thought it would be better, but I had difficulty simultaneously learning all of the different parts of electrical engineering.

    However, I would take it further than you when it comes to education. Einstein said “The only thing that interferes with my learning is my education.” 99% of everything I learned that I consider significant I learned outside of a classroom setting. This includes the research on facial features I once told you about.

    There is one professor I remember from my undergrad at the large state school that I would never trade for anyone, not even any Nobel Winner, because he was amazingly talented in teaching.

    And correct me if I’m wrong, but the vast majority of Ivy League applicants already have mostly perfect scores, so they have to weed out in other ways.

  178. Quixotic Reimagining of Standardized Tests (Part 1) | BetaWorldProblems Says:

    […] some convincing arguments for using standardized tests more in college admissions (Pinker, then Aaronson). Despite the imperfections of tests, they argue, the alternatives are likely to be less fair and […]

  179. Lots More Random Stuff » Undivided Looking Says:

    […] Then again, maybe we shouldn't let the people running them now be in charge […]